Unit 1 Exam

Pataasin ang iyong marka sa homework at exams ngayon gamit ang Quizwiz!

12-20 yrs

(Adolescence) Identity V role diffusion

20-35yrs

(Adulthood) Intimacy V Isolation

65+ yrs

(Late Adulthood) Integrity V despair

3-6 yrs

(preschool) Initiative V Guilt

referred pain

(referring to pain) pain that is felt at a location other than its origination; when one physician sends (refers) a client to another physician or specialist.

What is the order for putting on personal protective gear?

*REMEMBER PUT YOUR GOWN ON MOM SO WE CAN GO GET YOUR GLOVES 1. Gown 2. Mask 3. Goggles 4. Gloves

WHAT IS THE RULE: NEVER SEND A PATIENT TO SURGERY ANTICOAGULANT

-- Low platelets -- High PT/ PTT --Coumadin/ Heparin ----TURP --- PT 20 ( report to the doctor)

RULE: WHAT YOU CAN USE WHEN PATIENT GOING TO SURGERY AND WHICH MED YOU CAN HOLD

---Aqua Mephyton = Vitamin K = given coumadin going to surgery ----Morphine SO4 40mg IV stat for MI ( no more than 40mg) ----When someone have low K+ hold the digoxin ----- Immunocompromised ----- PCP (pneumocyst carini pneumonia) Peds nurse float to --------- Leukemia or chemo

CONDITIONS REQUIRED SEIZURE PRECAUTIONS

1. Cirrhosis encephalopathy 2. PIH (pregenancy induced hypertension) 3. DT'S (delirium tremors) 4. ICP 5. CVA 6. Meningitis 7. Brian surgery 8. head trauma

INTERVENTION FOR PREVENT ASPIRATION

1. HOB up to eat 2. On right side after eating 3. Suction avialable 4. Side rails up 5. Bed in low position 6. Call bell w/ patient reach

RULE: TRANSFERRING PEDS PATIENT TO MS (MED SURGE)

1. Oldest child 1st 2. No communicable diseases 3. Not immunocompromised 4. No teaching needed]

DON'T TRANSFER WHICH PATIENT

1. Patient which are UNSTABLE 2. Unknown diagnosis 3. Patient whose condition is made worse by stress

Who should get the chicken pox vaccine

1. Unvaccinated children 12months and older 2.Susceptible non pregnant women 3. Susceptible persons living in a dorm or jail 4. International travelers

WHICH PATIENT NEED TO ISOLATE

1.. Patient with/ night sweats and temp w/cough (TB). 2. Headache and stiff neck ( Meningitis). 3. Adult w/rash or blisters only on one side (Shingles). 4. Any patient w/ S&S of infection: 1ST ACTION ISOLATE

Who shouldn't get the Chicken pox vaccine

1.Anyone recieving high doses of steriods 2. Pregnant women 3. A allergy to neomycin or gelatin 4.Anyone w/ an active infection 5. Immuncomprimised pt's

How would you take off Personal protective gear?

1.Gloves 2.Goggles 3.Gown 4.Mask

What should the nurse immediately do if she stuck by a used needle?

1.Properly dispose of the needle, remove gloves 2.Wash your hands 3.Alert your manager or report to the health dept

TRANSFERRING PATIENT BETWEEN WARDS

1.Recieve ward must be familar w/ the disease/ treatment 2. Be alert for gender specific wards

RULE: DISCHARGE DURING AN EMERGENCY

1.Select patient with chronic stable conditions (EX: Pressure ulcer = Stable chronic condition) 2. Do not discharge acute surgical patients

Normal range respriatory rates adult Men

12-18

Normal blood pressure parameters for adults

120/80

Normal range respriatory rates adult

16-20

RULE: MED ERROR, PT INJURY OR ATTEMPT SUICIDE

1St provide care 2nd notify MD ( medical doctor

Normal range oral temperature

35.5-37.5 C (95.5-99.5 F)

Rabies pt's would need

5 shots Immunoglobulins are given but if there showing symtoms it's too late

Normal pulse rate for adults

60-80 BPM ( beats per minute)

19. If a nurse experiences a problem reading a physician's medication order, the most appropriate action will be to: A. Call the physician to verify the order B. Call the pharmacist to verify the order C. Consult with other nursing staff to verify the order D. Withhold the medication until the physician makes rounds

A

25. Nursing theories provide nurses with perspectives to: A. Analyze client data B. Predict situations C. Formulate legislation D. Link science to nursing (Metaparadigm: Environment, Health, Person, Nursing)

A

31. A client complains of chest pain, When assessing the pain, you decide its origin is cardiac- rather than respiratory or GI -when it: A. Does not occur with respiratory variations B. Is peripheral and may radiate to the scapular region C. Is aggravated by inspiratory movements D. Is non-radiating and occurs during inspiration

A

32. A client with a tracheostomy has thick tenacious secretions. To maintain the airways, the most appropriate action for the nursing includes: A. Tracheal suctioning B. Oropharyngeal suctioning C. Nasotracheal suctioning D. Orotracheal suctioning

A

35. A senior student nurse delegates the task of intake and output to a new nursing assistant. The student will verify that the nursing assistant understand the task of I an O when the nursing assistant states: A. "I will record the amount of all voided urine" B. "I will not count liquid stools as output" C. "I will not record a cafe mocha as intake" D. "I will notate perspiration and record it as small or large amount"

A

43. In order for a student to avoid a data collection error the student should: A. Assess the client and, if unsure of the finding, ask the faculty to assess the client B. Review his or her own comfort level and competency with assessment skills C. Ask another student to perform the assessment D. Consider the diagnosis should be actual, potential, or risk

A

What is the most dangerous type of Hep?

A

Which of the following assures clients that they will receive quality care from a competent nurse? A) Standards of care B) Nurse Practice Act C) Accreditation certification D) National council licensure

A

An older adult recently diagnosed with a urinary tract infection displays a sudden onset of confusion. She most likely is experiencing: A) Delirium B) Dementia C) Depression D) Social isolation

A Delirium is a potentially reversible cognitive impairment that often has a physiological cause such as a urinary tract infection. Onset is typically sudden. Dementia is characterized by a gradual, progressive, irreversible cerebral dysfunction that leads to a decline in the ability to carry out the activities of daily living. Depression typically has an insidious onset, and the person displays a lack of interest or pleasure in living. Thinking and perception remain intact except in severe cases. Social isolation is characterized by reduced interaction with others. It may occur by choice or in response to conditions that decrease the ability or opportunity to interact with

The nurse says to the client, "We've talked a lot about your medications, but let's look more closely at the trouble you're having in taking them on time." The nurse is using the therapeutic technique of: A) Focusing B) Clarifying C) Paraphrasing D) Providing information

A Focusing is used to center attention on key concepts or elements in a message. Clarifying gives the client a chance to be more specific or provide more information. Paraphrasing means restating another's message briefly in one's own words. The nurse is not providing information here.

Theories that are broad and complex are: A) Grand theories B) Descriptive theories C) Middle-range theories D) Prescriptive theories

A Grand theories are described as broad and complex. Middle-range theories are limited in scope, less abstract, address specific phenomena or concepts, and reflect practice. Descriptive theories describe phenomena, speculate as to why the phenomena occur, and describe the consequences of phenomena. Prescriptive theories address nursing interventions and predict the consequence of a specific intervention.

Leininger's theory of cultural care diversity and universality specifically addresses: A) Caring for clients from unique cultures B) Understanding the humanistic aspects of life C) Identifying variables affecting a client's response to a stressor D) Caring for clients who cannot adapt to internal and external environmental demands

A The goal of Leininger's theory is to provide the client with culturally specific nursing care, in which the nurse integrates the client's cultural traditions, values, and beliefs into the plan of care.

The nurse says to the client, "We've talked a lot about your surgery and the implications for you when you go home. Let's discuss some of the exercises you can do." This is an example of: A) Focusing B) Clarifying C) Summarizing D) Providing information

A The nurse's statements depict the therapeutic communication technique of focusing. Focusing is used to center attention on key elements or concepts of a message. Focusing helps guide the direction of the conversation. Summarizing is a concise review of key aspects of an interaction. It brings closure to a conversation. Clarifying is used to check whether the listener's understanding is accurate. Instead of restating the message as in paraphrasing, the nurse asks the other person to rephrase the message, explain further, or give an example of what the person means. Providing relevant information tells other people what they need or want to know so that they can make decisions, experience less anxiety, and feel safe and secure.

Answer: C Pallor would appear as yellowish brown in brown-skinned people. Pallor would manifest as bluish skin in light-skinned people. Pallor would appear as ashen gray skin in black-skinned people. Shiny skin indicates edema.

A common abnormality encountered during inspection of the skin is pallor. Pallor is easily seen in the face, mucosa of the mouth, and nail beds. How would pallor appear in a brown-skinned client? A) As shiny skin B) As bluish skin C) As yellowish skin D) As ashen gray skin

What kind of diet is needed for a PT with hepatitis A

A diet high in calories and low fat

Answer: A, C Clients being discharged home need education regarding how to take their medication and when to call their health care provider. There is not enough information here to determine if options 2 and 4 are appropriate, although hand hygiene after toileting is always important.

A nurse is planning a client's discharge from a subacute care unit to home. Education should be provided on which of the following topics? (Select all that apply.) A) Medication administration B) Stress reduction techniques with blood pressure assessment C) Circumstances in which the client should call the health care provider D) Hand-washing hygiene when assisting with transfer to the bathroom

Answer: B A counselor helps clients identify and clarify health problems and choose appropriate courses of action to solve those problems. An educator helps the community gain greater skills, including through the presentation of educational programs. A collaborator is an individual who engages in a combined effort with other individuals to develop a mutually acceptable plan that will achieve common goals.

A nurse is practicing in an occupational health setting. There are a large number of employees who smoke, and the nurse designs an employee assistance program for smoking cessation. This is an example of which nursing role? A) Educator B) Counselor C) Collaborator D) Case manager

Answer: D Taking blood pressure measurements is illness prevention. Health promotion includes activities like exercise classes. Secondary care is often known as traditional care. It would include rehabilitation after a stroke in an individual with a history of elevated blood pressure.

A nurse volunteers to take blood pressure measurements after church services. This is an example of which level of health care service? A) Secondary care B) Restorative care C) Health promotion D) Illness prevention

When dealing w/ a Droplet percautions pt they require

A private room but the door can remain open Use standard percautions but a wear a MASK WHEN YOUR W/IN 3FT OF THE PT *SPIT ZONE

Answer: D Change must be perceived as advantageous, compatible with existing values, and easily adaptable to be successful and accepted. Up-front cost, managerial framework, building plans, contractors, compliance with building codes, and regulations for governmental agencies are all incorporated in proposals but do not provide convincing reasoning that leads to change.

A proposal written by a community-based nurse for a new, higher quality older adult care center will have increased probability of acceptance if the proposal includes: A) All building plans and a list of contractors to complete the job B) Compliance with the codes and building requirements of local government agencies C) The up-front cost and managerial framework of the new older adult center D) Description of how advantageous, realistic, compatible, and adaptable the change will be when implemented

What is a GUAIAC test?

A test to decect fecal occult blood assess for hidden blood in stool

Answer: B A theory is a set of concepts, definitions, relationships, and assumptions that explains a phenomenon. Theories do not formulate legislation, measure nursing functions, or reflect any domain of nursing practice.

A theory is a set of concepts, definitions, relationships, and assumptions that: A) Formulates legislation B) Explains a phenomenon C) Measures nursing functions D) Reflects the domain of nursing practice

The nursing assessment is which phase of the nursing process? A) First B) Second C) Third D) Fourth

A) First

The nurse asks the client whether the client has any allergies. This is an example of: A) Health history data B) Biographical information C) History of present illness D) Environmental history data

A) Health history data Known allergies are a part of historical data. Biographical data include age, address, occupation, work status, marital status, course of health care, and insurance. The history of the present illness includes when the symptoms began, whether they began suddenly or gradually, whether they come and go, and other information about the illness. The environmental history includes data about the client's home and working environments.

The nurse asks a client, "Ms. Neil, describe for me your typical diet over a 24-hour day. What foods do you prefer? Have you noticed a change in your weight recently?" This series of questions would likely occur during which phase of a client interview?

A) Working

Answer: B APNs are generally the most independently functioning nurses. An APN can work in a primary, acute, or restorative care setting. The setting may be a private, public, or university facility. The APN may function as a clinician, educator, case manager, consultant, or researcher.

Advanced practice nurses (APNs) generally: A) Work in acute care settings B) Function independently C) Function as unit directors D) Work in the university setting

Answer: A Care provider is a staff position, a nurse who provides direct care. The nurse specialist has clinical expertise in a specific area. The nurse practitioner has advanced training in assessment and pharmacology and is able to provide health care in specific settings. The case manager has additional experience and is able to coordinate activities of other members of the health care team.

An APN is the most independently functioning of all professional nurses. All of the following are examples of a clinically focused APN except: A) Care provider B) Case manager C) Nurse specialist D) Nurse practitioner

Answer: B Surgical clients are the client population of interest (P) in the PICO (population, intervention, comparison, outcome) question. Chlorhexidine use is the comparison of interest, and povidone-iodine use is the intervention of interest. The operating room nurse is not an element of the PICO question.

An operating room nurse is talking with colleagues during a meeting and asks, "I wonder if we would see fewer wound infections if we used chlorhexidine instead of povidone-iodine to clean the skin of our surgical clients? In this example of a PICO question, the P is: A) Povidone-iodine use B) Surgical clients C) Chlorhexidine use D) Operating room nurses

Answer: D As a science, nursing draws on scientifically tested knowledge applied in the practice setting.

As an art, nursing relies on knowledge gained from practice and reflection on past experiences. As a science, nursing relies on: A) Experimental research B) Nonexperimental research C) Physician-generated research D) Scientifically tested knowledge

13. You notice that a teenager has an irregular pulse. The best action you should take includes: A. Read the history and physical B. Assess the apical pulse rate for one full minute C. Auscultate for strength and depth of pulse D. Ask if the client feels any palpitations or faintness of breath

B

18. Nurses are legally required to document medications that are administered to clients. The nurse is mandated to document: A. Medication before administering it B. Medication after administering it C. Rationale for administering the medication D. Prescriber's rationale for prescribing the medication

B

46. Consultation occurs most often during which phase of the nursing process? A. Assessment B. Planning C. Implementation D. Evaluation

B

A nurse who uses critical thinking in the decision-making process to provide effective quality care to individuals is known as: A) An advanced care nurse B) A clinical decision maker C) A member of a multidisciplinary practice D) An evidence-based practitioner

B

Advanced practice nurses (APNs) generally: A) Work in acute care settings B) Function independently C) Function as unit directors D) Work in the university setting

B

The licensure and practice of nursing is regulated by: A) The NCLEX-RN B) The Nurse Practice Act C) The certification examination D) The ANA Congress for Nursing

B

Which of the following clients should be cared for in an extended care facility with skilled nursing? (Select all that apply.) A) Client who had a stroke, can talk, and has lost bowel and bladder control B) Severely brain injured client on a ventilator who is receiving intravenous medications C) Client with Alzheimer's disease who is abusive, combative, and a threat to self and others D) Young child who recently had a spinal cord injury and is living with quadriplegia and needs to learn a new way of life

B, D Extended care encompasses intermediate medical, nursing, or custodial care for clients recovering from acute illness or clients with chronic illnesses or disabilities. Extended care facilities include intermediate care facilities and skilled nursing facilities.

1. Nursing is defined as a profession because nurses: A. Perform specific skills B. Practice autonomy C. Utilize knowledge from the medical discipline D. Charge a fee for services rendered.

B. Practice autonomy

WHen should you use standard percautions

BLOOD ALL BODY FLUIDS NON INTACT SKIN MUCOUS MEMBRANES

When is a pt w/measles most contagious?

Before the rash appears

Answer: B Hypertension is often asymptomatic until pressure is very high. Headache (usually occipital), facial flushing, nosebleed, and fatigue are common symptoms of hypertension. Restlessness and dusky or cyanotic skin that is cool to the touch, dizziness, mental confusion, and mottled extremities are all signs and symptoms of hypotension. Unexplained pain and hyperactivity are very vague complaints.

Besides high blood pressure values, what other signs and symptoms may the nurse observe if hypertension is present? A) Unexplained pain and hyperactivity B) Headache, flushing of the face, and nosebleed C) Dizziness, mental confusion, and mottled extremities D) Restlessness and dusky or cyanotic skin that is cool to the touch

What should you wash the toilet seat with

Bleach

What colors indicate blood in the stool when doing GUAIAC test

Blue Blue-green GREEN means negative

10. A nurse records that the client stated his abdominal pain is worse more than last night. This is an example of: A. PIE documentation B. SOAP documentation C. Narrative charting D. Charting by exception

C

14. A postoperative client is breathing rapidly. You should immediately: A. Call the physician B. Count the respirations C. Assess the oxygen saturation D. Ask the client if they feel uncomfortable

C

15. When assessing the BP of a school-age child using a normal-size adult cuff will affect the reading and produce a value that is: A. Accurate B. Indistinct C. Falsely low D. Falsely high

C

23. A student nurse employed as a nursing assistant may perform care: A. Learned in school B. Expected of a nurse at that level C. Identified in the hospital's job description D. Requiring technical rather than professional skills

C

29. A nurse floats to a busy surgical unit and administers a wrong medication to a client. This error can be classified as: A. A poisoning accident B. An equipment-related accident C. A procedure-related accident D. An accident related to time management

C

37. A 4 year old pediatric client resists going to sleep. In order to assist this client, the best action to take would be: A. Adding a daytime nap B. Allowing the child to sleep longer in the morning C. Maintaining the child's home sleep routine D. Offering the child a bedtime snack

C

41. The nursing process organizes your approach to delivering nursing care. In order to provide care to your clients, you will need to incorporate nursing process and: A. Decision making B. Problem solving C. Interview process D. Intellectual standards

C

Evidence-based practice is defined as: A) Nursing care based on tradition B) Scholarly inquiry embodied in the nursing and biomedical research literature C) A problem-solving approach to clinical practice based on best practices D) Quality nursing care provided in an efficient and economically sound manner

C

The role of the nurse in planning care for a culturally diverse population can best be described as: A) Focusing only on the needs of the client, ignoring the nurse's beliefs and practices B) Including care that is culturally congruent with the nursing staff based on predetermined criteria C) Providing care while remaining aware of one's own bias and focusing on the client's individual needs rather than the staff's practices D) Blending the values of the nurse that are for the good of the client and minimizing the client's individual values and beliefs during care

C

Visual acuity declines with age. Presbyopia is a progressive decline in: A) Ability to see in darkness B) Adaptation to abrupt changes from dark to light C) Ability of the eyes to accommodate for close, detailed work D) Ability to distinguish between blues and greens and among pastel shades

C

When caring for the older adult, it is important to: A) Repeat oneself often because older adults are forgetful. B) Be aware that older adults are no longer interested in sex. C) Treat the client as an individual with a unique history of his or her own. D) Disregard the older adult's experiences because older people are too old-fashioned to be of value today.

C

A client's family member wipes her eyes as she cries at the loss of a loved one and says, "It's no big deal. I mean, we all have to die sometime, right?" The nurse is engaging in metacommunication when the nurse responds: A) "You are taking this really well." B) "You are exactly right. Dying is inevitable." C) "Losing a loved one can be really difficult. It looks like you're pretty upset. I'd like to help." D) "Let's not talk about it. That will help you feel better. After all, God won't give you more than you can handle."

C Metacommunication uncovers the deeper message beneath what is being overtly said. Option 3 would be the most therapeutic response by the nurse. In options 1 and 2, the nurse is not attending to the nonverbal cues in these responses but sounds oblivious to the real feelings being expressed by the family member. The nurse should allow the client to verbalize those feelings.

What is the most common cause of ATB associated diarrhea

C Diff( Clostridium difficle)

WHICH PATIENT CAN KEEP IN THE SAME ROOM

COPD + Negative pressure room = Keep together

What type of job should a person with hepatitis never have

Chef

What precaution are needed for Scabies pt

Contact which are gloves and gown * Toes, wet, warm, spots

How is c diff transmitted?

Contact you touch it you get it the nurse would touch something contaimanated then transfer it by direct pt to pt contact

How is Salmonella transmitted

Contaminated food and water

Which of the following statements by the nurse would be nontherapeutic and tend to block communication? (Select all that apply.) A) "You look sad today." B) "Why are you so nervous?" C) "If I were you, I'd have the surgery." D) "I'm sure the test results will come out fine." E) "Tell me what it's like to live with dizziness."

Correct Answer(s): A, B, C, D Nontherapeutic statements hinder communication. False reassurances block communication and destroy trust. Asking for explanations can be interpreted as "testing" and can cause resentment, insecurity, and mistrust. Giving personal opinions takes the decision making away from the client.

16. You are caring for a client who has diabetes complicated by kidney disease. You will need to make a detailed assessment when administering medications because this client may experience problems with: A. Absorption B. Biotransformation C. Distribution D. Excretion

D

A client with diabetes mellitus who takes daily insulin injections is scheduled for surgery the next day. The client is to take nothing by mouth (NPO status) after midnight. The nurse questions whether insulin should be given the morning of surgery. This is an example of: A) Problem solving B) Previous experience C) Clinical practice guideline D) Scientifically based clinical judgment

D

Although the older adult's libido does not decrease, changes occur in sexual activity. Which of the following is correct with regard to these changes? A) The need to touch and be touched is decreased. B) The sexual preferences of older adults are not as diverse. C) Physical changes usually will not affect sexual functioning. D) Frequency of and opportunities for sexual activity may decline.

D

Cultural competence is the process of: A) Learning about the large number of cultures B) Developing motivation and commitment to caring C) Influencing the treatment and care of clients D) Acquiring specific knowledge, skills, and attitudes

D

Culture strongly influences pain expression and need for pain medication. However, cultural pain: A) Is not expressed verbally or physically B) Is expressed only to others of like culture C) Is more intense, thus necessitating more mediation D) May be suffered by a client whose valued way of life is disregarded by practitioners

D

The examination for the registered nurse (RN) licensure is exactly the same in every state in the United States. This examination: A) Guarantees safe nursing care for all clients B) Ensures standard nursing care for all clients C) Ensures that honest and ethical care is provided D) Provides a minimal standard of knowledge for practice

D

The nurse found that using tympanic thermometers was quick, easy, and yielded temperatures as reliable as those obtained using oral thermometers. This finding represents: A) Primary care B) Critical thinking C) Competency testing D) Evidence-based practice

D

The nurse practices nursing in conformity with the code of ethics for professional registered nurses. This code: A) Improves self-health care B) Protects the client from harm C) Ensures identical care to all clients D) Defines the principles by which nurses' provide care to their clients

D

Who acted to decrease mortality by improving sanitation in the battlefields, which resulted in a decline in illness and infection? A) Dorothea Dix B) Lillian Wald C) Clara Barton D) Florence Nightingale

D

1. Nursing's paradigm includes: A) Health, person, environment, and theory B) Concepts, theory, health, and environment C) Nurses, physicians, models, and client needs D) The person, health, environment/situation, and nursing

D Nursing's paradigm includes four linkages: the person, health, environment/situation, and nursing.

Which of the following is objective information to be recorded in the client's medical record? A) Anxious over upcoming test. B) Increasing stress over past 2 months. C) Performs breast self-examination monthly. D) Expelled 1 tablespoon of yellow sputum.

D) Expelled 1 tablespoon of yellow sputum. Objective data are measurable data. Options 1, 2, and 3 describe data that cannot be measured by the nurse but depend on the client's reports; thus they are subjective data.

What should you do if the c diff is linked to an ATB

DISCONTINUE THE ATB

What should you teach pt's to wear for protect themselves against Lyme disease

Deet spray

Answer: D The nurse may delegate vital signs measurement to unlicensed assistive personnel when the client is in stable condition, the results are predictable, and the technique is standard. The preoperative client is the only client listed who meets these guidelines.

Delegation of some tasks may become one of the decisions the nurse will make while on duty. For which of the following clients would it be most appropriate for unlicensed assistive personnel to measure the client's vital signs? A) A client who recently started taking an antiarrhythmic medication B) A client with a history of transfusion reactions who is receiving a blood transfusion C) A client who has frequently been admitted to the unit with asthma attacks D) A client who is being admitted for elective surgery who has a history of stable hypertension

chronic

Disease develops slowly and persist over long periods

What should a Contact Precaution pt have

Disposable dinnerware GLOVES/GOWN ALWAYS need dedicated equipment and disinfect room

What are char of Pertussis( whooping cough)

Dry cough

Answer: D Kyphosis (hunchback) is an exaggeration of the posterior curvature of the thoracic spine and is common in older adults. Lordosis (swayback) is increased lumbar curvature. Scoliosis is lateral spinal curvature. Hypotonic muscle has little tone and feels flabby, usually because of atrophy of muscle mass.

During general inspection of the musculoskeletal system of an older client, the nurse notes kyphosis. Kyphosis is: A) Lateral spinal curvature B) Loss of or decrease in muscle tone C) Increased lumbar curvature D) Exaggeration of the posterior curvature of the thoracic spine

42. Concept mapping is one way to: A. Connect concepts to a central subject B. Relate ideas to client health problems C. Challenge a nurses thinking about client needs and problems D. Graphically display ideas by organizing data E. All of the above

E

What pt's need to use Standard, Airbourne, Contaction and Droplet Precautions (SAC D)

EBOLI MARBAR CRIMEAN CONGO

Answer: C The domain contains the subject, central concepts, values and beliefs, phenomena of interest, and the central problems of the discipline. A paradigm is a model that explains the linkage of science, philosophy and theory that is accepted and applied by the discipline.

Each science has a domain, which is the perspective of the discipline. This domain: A) Represents the recipients of the benefits of the science or discipline B) Is a model that explains the linkage of science, philosophy, and theory that is accepted and applied by the discipline C) Describes the subject, central concepts, values and beliefs, phenomena of interest, and central problems of the discipline D) Is a dynamic state of being in which the developmental and behavioral potential of the individual is realized to the fullest

turgor

Elasticity of the skin

American Nurses Association's (ANA's) Code of Ethics

Ethical principles for professional nursing practice in a clinical setting are guided by the principles of conduct that are written as the:

Answer: D The result of theory-generating or theory-testing research is to increase the knowledge base of nursing. As these research activities continue, clients become the recipients of evidence-based nursing care.

Evidence-based nursing practice is the end result of: A) Prescriptive theory B) Use of practical knowledge C) Application of theoretical knowledge D) Theory-generating and theory-testing research

Identify the type of precaution required

Examples: Airborne, droplet, contact or standard

How is it spread

Fecal/oral

Where can you find Salmonella

Feces of infected people or animals

Who needs Standard Contact and Droplet precautions

Glanders Lassa

What should you wear when drawing blood

Gloves

What is warn when suctioning?

Gloves, Goggles and a face shield

What should you wear if blood is spurting?

Gloves, gown, goggles and mask EVERYTHING

CONTACT PRECAUTIONS

Gown and gloves and disinfectant all the equipments

What pt's would be ok for a pregnant nurse to care for

HIV HEP B,C Cytmegalovirus

Answer: D Healthy People 2010 was established to create ongoing health care goals, including increasing life expectancy and quality of life, and eliminate health disparities through improved delivery of health care services. Gathering information, assessing needs, and developing and implementing public health policies are steps in achieving the goals set forth by Healthy People 2010.

Healthy People 2010's overall goals are to: A) Assess the health care needs of individuals, families, or communities B) Develop and implement public health policies and improve access to care C) Gather information on incident rates of certain diseases and social problems D) Increase life expectancy and quality of life and eliminate health disparities

MISC. FACTS

If age is in the question (?). Consider growth and development When they given output over time always calculate to the hr. EX: urine over time (60-75cc per hr) OR chest tubes < 100cc/hr FOR HEMORRHAGE: Always chech pulse no BP Eliminate adjective in ? BUN (blood urea nitrogen) is not a measure of kidney function only serum creatinine. NEVER give drug with conflicting actions. WHEN explaining conditions to families or testifying in trial: DO NOT use medical jorgon, use simple direct answer. CALL dr. for abnormal situations not what is expected from disease. Add or remove terms from this set

Answer: B In Healthy People 2010, the assurance role of public health is defined as making essential community-wide health services available and accessible. In Healthy People 2010, public development and implementation refer to the role of health professionals in providing leadership in development of policies that support the population's health. Population-based public health programs focus on disease prevention, health promotion, and health protection. A healthy environment for each individual, family, and community is the overall goal of Healthy People 2010.

In Healthy People 2010, assurance refers to the role of public health in: A) Providing disease prevention, health protection, and health promotion B) Making essential community-wide health services available and accessible C) Providing leadership in developing policies that support the population's health D) Achieving a healthy environment for each individual, family, and community

Answer: C Wheezes are adventitious breath sounds that are high-pitched, continuous musical sounds, such as a squeak heard continuously during inspiration or expiration. Pleural friction rub has a dry, grating quality and is heard best during inspiration. Crackles can be fine, high-pitched, short, interrupted crackling sounds; moist, low sounds in the middle of inspiration; or coarse, loud, bubbly sounds. Rhonchi are loud, low-pitched, rumbling, coarse sounds heard during inspiration.

In assessing the client's lungs the nurse hears adventitious breath sounds that are high-pitched, continuous musical sounds, such as a squeak heard continuously during inspiration or expiration, usually louder on expiration. These adventitious breath sounds are known as: A) Crackles B) Rhonchi C) Wheezes D) Pleural friction rub

Answer: C Resonance is the low, hollow sound of normal lungs. Hyperresonance can be heard over emphysematous lungs as a booming sound. Tympany is the high-pitched, drumlike sound heard over a gastric air bubble. Dullness is the soft, thudlike sound that is heard over dense organ tissue.

In assessing the client's lungs, the nurse notes that the lungs are normal upon percussion. This means that the nurse detected: A) Dullness B) Tympany C) Resonance D) Hyperresonance

WHEN CAN INFECTED PATIENT LEAVE THEIR ROOM

Infected patient can leave their rooms as long as they wear the same protective garments out of their room that we wear when we are going (entering in ) into their room.

What are characteristics of Measles (Rubeola)

Kopliks spots

Answer: A The goal of Leininger's theory is to provide the client with culturally specific nursing care, in which the nurse integrates the client's cultural traditions, values, and beliefs into the plan of care.

Leininger's theory of cultural care diversity and universality specifically addresses: A) Caring for clients from unique cultures B) Understanding the humanistic aspects of life C) Identifying variables affecting a client's response to a stressor D) Caring for clients who cannot adapt to internal and external environmental demands

What is the tx for Scabies

Lindane Crotmiton Permethrine

Which pt's can't the pregnant nurse care for

MEASLES GERMAN MEASLES PARAVIRUS B19 VARICELLA CHICKEN POX RADIO ACTIVE ISO TOPE HANGING CHEMO MEDS

When should you use Airborne Precautions

MY CHICKEN HEZ TB Measles(RUBEOLA) Chicken pox (VARICELLA) Herpes zoster(SHINGLES) IMMUNOCOMPROMISIED AND DISSEMINATED TB

What are the characteristics of Chicken pox Varicella?

Maculopapular rash and vesicular scabs

Answer: C The first level of Maslow's hierarchy of needs includes the need for air, food, and water—basic elements of survival. Love and belonging are on the second level, esteem and self-esteem are on the fourth level, and self-actualization is the final level.

Maslow's hierarchy of needs is useful to nurses, who must continually prioritize a client's nursing care needs. The most basic or first-level needs include: A) Self-actualization B) Love and belonging C) Air, water, and food D) Esteem and self-esteem

Answer: D Middle-range theories are limited in scope, less abstract than grand theories, address specific phenomena or concepts, and reflect practice. Grand theories are described as broad and complex. Prescriptive theories address nursing interventions and predict the consequence of a specific nursing intervention. Descriptive theories describe phenomena, speculate as to why the phenomena occur, and describe the consequences of phenomena.

Mishel's theory of uncertainty in illness focuses on the experience of clients with cancer who live with continual uncertainty. The theory provides a basis for nurses to assist clients in appraising and adapting to the uncertainty and illness response and can be described as: A) A grand theory B) A descriptive theory C) A prescriptive theory D) A middle-range theory

What are the S/S for Botulism

N, V, D and Abd pain

If aptient is going to surgery do not give them

NSAIDS/ASA, HEPARIN, COUMADIN GARLIC, GINKGO BILOBA, GINSENG AND VITAMIN K

After getting the Varicella vacc who should you teach pt's to avoid

Newborns pregnant women Immuncompromised pts for 1 MONTH

GENERAL RULES FOR VACCINES

No vaccine if : patient has temp > 101 on antibiotics For exposure to any infection/ enlarged lymph nodes: immunoglobin given----- for pain and discomfort Misc Facts

What should you wear when taking a pt's VS?

Nothing unless the pt is in isolation

Assess the client's point of view and prepare to articulate this point of view.

Nurses agree to be advocates for their patients. Practice of advocacy calls for the nurse to:

Answer: A A nurse who is new to practice has not developed the experience required for research but can begin at the less complicated level of data collection. An experienced researcher is more qualified to identify problems for formal research, although input from all levels of nursing is valuable. Nurses with doctoral-level training are typically prepared for obtaining financial backing. An American Nurses Association position paper cites a master's degree as qualification for implementing research-based change in nursing practice.

Nurses who are new to practice can best contribute to nursing research by: A) Assisting with data collection B) Identifying clinical problems in nursing C) Obtaining financial backing and public interest D) Implementing research-based change in nursing practice

What other ATB might be prescribed to treat c diff

Oral Flagyl and and Vancomycin

What kind of room would an Airborne precaution pt need

PRIVATE NEGATIVE PRESSURE ROOM *REMEMBER PN

What are the major ATB associated w/ c diff

Pencillins Cephalosporins Clindamycin

What are the char. of German measles( Rubella)

Pink red maculopapules

Infected Air-borne precaution

Private room with negative pressure

patient what kind of room they should stay

Private rooms or patient with same organisms

Answer: A A positive benefit of a professional nursing staff is a decreased length of stay. The diagnosis-related group has greater influence on the rate of readmission. The ancillary personnel need to remain so that registered nurses can spend the necessary time to assess and manage clients. Nosocomial infections decrease with a professional nursing staff.

Recent research provided evidence that a professional nursing staff affects health care financing. These results indicated that the positive benefit of a professional nursing staff is: A) Decreased length of stay B) Decreased rate of readmission C) Increased rate of nosocomial infections D) Decreased need to hire ancillary personnel

What are the Ni for Pertussis?

Rest, warm, humid air

What is the two diseased caused by ticks

Rocky mountain fever and Lymes disease

Who needs Standard, Airbourne and Contact precautions

Small pox

Answer: A Lying in the supine position with the ipsilateral arm behind the head helps the breast tissue to flatten evenly against the chest wall. The other options do not allow the tissue to spread on the chest wall.

So that breast tissue will be spread evenly over the chest wall during an examination, the nurse asks the client to lie supine with: A) The ipsilateral arm behind the head B) Hands clasped just above the umbilicus C) Both arms overhead with palms upward D) The dominant arm straight alongside the body

A review of systems (ROS) is based on information obtained from the client during the interview. This information is an example of ______________ data.

Subjective

Answer: C Substance abusers avoid health care for fear of judgmental attitudes by health care providers and concern about being turned in to the criminal authorities. Options 1, 2, and 4 are not primary concerns that result in avoidance of health care.

Substance abusers frequently avoid health care providers because of: A) Fear of the cost of health care B) Fear of institutions and people C) Fear of being turned in to the criminal authorities D) Fear of being without the recreational drug of choice

Answer: D Therapies such as tepid water or alcohol sponge baths should be avoided because they lead to shivering, which stimulates body heat. Antipyretics, not analgesics, are the medications that lower body temperature.

The client has an oral temperature of 39.2° C (102.6° F). What are the most appropriate nursing interventions? A) Provide an alcohol sponge bath and monitor laboratory results. B) Remove excess clothing, provide a tepid sponge bath, and administer an analgesic. C) Provide fluids and nutrition, keep the client's room warm, and administer an analgesic. D) Reduce external coverings and keep clothing and bed linens dry; administer antipyretics as ordered.

Answer: D Adduction is movement toward the body. Abduction is movement away from the body. Flexion is movement that decreases the angle of the joint, whereas extension is movement that increase the angle of the joint.

The client is being assessed for range of joint movement. The nurse asks the client to move the arm toward the body to evaluate: A) Flexion B) Extension C) Abduction D) Adduction

Answer: B Rhonchi are loud, low-pitched, rumbling, coarse sounds heard most often during inspiration or expiration. Crackles are moist sounds heard during inspiration that are not cleared with coughing. Normal lungs produce no sounds such as that described. Wheezes are high-pitched continuous muscles sounds such as a squeak heard continuously during inspiration and expiration.

The client's respiratory assessment reveals loud, low-pitched, rumbling, coarse sounds heard during inspiration and expiration. The nurse interprets these sounds as: A) Normal B) Rhonchi C) Crackles D) Wheezes

Civil

The client's right to refuse treatment is an example of _________ laws.

American Nursing Association

The code of ethics for nurses is composed and published by

Answer: C Healthy People 2010, a federal document, outlines goals for the public. Notes on Nursing set forth Nightingale's first nursing philosophy. The Last Acts Campaign has developed standards and policies for end-of-life care. Nursing Principles and Practice 2010—current readings in journals are necessary for all nurses in practice.

The document that developed goals and objectives to meet the health of the public is known as: A) Notes on Nursing B) Last Acts Campaign C) Healthy People 2010 D) Nursing Principles and Practice 2010

Answer: D The RN licensure examination provides a minimum standard of knowledge for nurses. The examination cannot guarantee or ensure care for clients.

The examination for the registered nurse (RN) licensure is exactly the same in every state in the United States. This examination: A) Guarantees safe nursing care for all clients B) Ensures standard nursing care for all clients C) Ensures that honest and ethical care is provided D) Provides a minimal standard of knowledge for practice

steatorrhea

The excretion of abnormal quantities of fat with the feces owing to reduced absorption of fat by the intestine.

dental caries

The formation of cavities in the teeth by the action of bacteria; tooth decay.

Answer: D The scientific method is the foundation of research and the most reliable and objective of all methods of gaining knowledge. Experience, critical thinking, and evidence are not the foundation of research.

The foundation of research is which of the following? A) Evidence B) Experience C) Critical thinking D) Scientific method

Answer: D The anterior hypothalamus controls heat loss by initiating the mechanisms of sweating and vasodilation of blood vessels. Blood is redistributed to surface vessels (flushing of the skin) to promote heat loss, not heat retention. The posterior hypothalamus controls heat production by initiating the mechanisms of shivering, vasoconstriction of blood vessels, and reduction of blood flow to the skin and extremities.

The hypothalamus controls body temperature. The anterior hypothalamus controls heat loss, and the posterior hypothalamus controls heat production. What heat conservation mechanisms will the posterior hypothalamus initiate when it senses that the client's body temperature is lower than comfortable? A) Vasodilation and redistribution of blood to surface vessels B) Sweating, vasodilation, and redistribution of blood to surface vessels C) Vasoconstriction, sweating, and reduction of blood flow to extremities D) Vasoconstriction, reduction of blood flow to extremities, and shivering

Answer: A Secondary intervention includes disease prevention after a health issue has been identified. Primary intervention is prevention of a health problem that has not yet occurred in the community. Tertiary intervention occurs after a problem has occurred and aims at preventing long-term negative impacts or recurrences in a population.

The local health department received information from the Centers for Disease Control and Prevention that the flu was expected to be very contagious this season. The nurse is asked to set up flu vaccine clinics in local churches and senior citizen centers. This activity is an example of which level of prevention? A) Primary intervention B) Tertiary intervention C) Nursing intervention D) Secondary intervention

Answer: A An educator helps clients, families, and communities gain greater skills and knowledge to provide their own care. An advocate is someone who helps clients walk through the system, identifies services, and plans for accessing appropriate resources. A collaborator is an individual who engages in a combined effort with other individuals to develop a mutually acceptable plan that will achieve common goals. A case manager develops and implements a plan of care.

The local school has an increasing number of adolescent parents. The nurse works with the school district to design and teach classes about infant care, child safety, and time management. These are examples of which nursing role? A) Educator B) Advocate C) Collaborator D) Case manager

Answer: C Case management is a model of organizing care in which the case manager monitors, directs, and advises the nursing care personnel on specific care issues and the progress of a client. In team nursing, care might be provided by groups composed of registered nurses, licensed practical nurses, and possibly assistive personnel. Nursing process is used to plan the nursing care for a client. Interdisciplinary care is care provided by a team whose members come from a variety of disciplines.

The multidisciplinary care model used to move clients efficiently from admission to discharge is known as: A) Team nursing B) Nursing process C) Case management D) Interdisciplinary care

Answer: D This common adage embodies an abstract idea, and explaining it indicates the client's ability to perform abstract reasoning. Judgment involves comparison and evaluation of facts and ideas to understand their relationships and to form appropriate conclusions. Knowledge is understanding or awareness of information gained through learning or experience. Association involves finding similarities between concepts.

The nurse asks the client to interpret the saying, "Don't count your chickens before they're hatched." The client's response provides information about the client's: A) Judgment B) Knowledge C) Association D) Abstract reasoning

Answer: D The movement of the head and shoulders is controlled by cranial nerve XI, the spinal accessory nerve. The facial nerve innervates the face. The hypoglossal nerve innervates portions of the tongue. The trigeminal nerve is a sensory and motor nerve enervating the side of the face and jaw.

The nurse asks the client to shrug the shoulders and turn the head side to side against the resistance of the examiner's hand. These actions allow the nurse to evaluate which cranial nerve? A) VII—Facial B) V—Trigeminal C) XII—Hypoglossal D) XI—Spinal accessory

Answer: C A critical pathway is a multidisciplinary treatment plan with interventions prescribed within a structured framework. A discharge plan includes an assessment and anticipation of the client's needs. Medicare is a federal health insurance plan for those 65 years of age and older. Standard nursing care is the minimum care to be given to a client.

The nurse completes the standard orders on a client's first day postoperatively. The instrument that is used to coordinate the client's care is: A) A Medicare plan B) A discharge plan C) A critical pathway D) Standard nursing care

Answer: A The general survey focuses on general appearance and behavior, including gender and race, age, signs of distress, body type, posture, gait, hygiene and grooming, dress, affect, mood, and speech. The other actions are carried out in different parts of the assessment.

The nurse conducts a general survey of an adult client, which includes: A) Checking appearance and behavior B) Measuring vital signs C) Observing specific body systems D) Conducting a detailed health history

Answer: D All questionable blood pressure readings should be rechecked. Ensuring the client's safety is a necessary safeguard, because low blood pressure is generally accompanied by weakness. For the majority of people, low blood pressure (systolic pressure of 90 mm Hg or below) is an abnormal finding and should be reported. Giving a client orange juice may raise blood glucose level but is not recommended to elevate blood pressure. Ambulating a client with hypotension would not be following safety precautions.

The nurse finds that the systolic blood pressure of an adult client is 88 mm Hg. What are the appropriate nursing interventions? A) Check other vital signs. B) Recheck the blood pressure and give the client orange juice. C) Recheck the blood pressure after ambulating the client safely. D) Recheck the blood pressure, make sure the client is safe, and report the findings.

Answer: D Evidence-based practice draws on both research and clinical experience. Competencies are evidence that skills have been demonstrated. Critical thinking is the questioning thought process that nurses need to use in practice. Primary care is health care provided in the community by one caregiver who takes responsibility for managing a client's care.

The nurse found that using tympanic thermometers was quick, easy, and yielded temperatures as reliable as those obtained using oral thermometers. This finding represents: A) Primary care B) Critical thinking C) Competency testing D) Evidence-based practice

Answer: B Critical thinking involves analyzing the data, learning, and problem solving to come up with a course of action. Tradition limits the ability to learn new ways and overlooks what research has to offer. The advice of experienced practitioners may limit research because experience may mean doing things the same way they have been done for years. Using personal opinion overlooks the objective data that are available.

The nurse involved in scientific research effectively analyzes the information collected and determines a course of nursing action by: A) Depending on tradition B) Using critical thinking C) Seeking the advice of experienced practitioners D) Relying on personal perspective or opinion

Answer: C An advocate helps speak for the client, communicating the client's concerns and wishes to family and other caregivers. A caregiver assists in meeting all health care needs of the client, including taking measures to restore emotional, spiritual, and social well-being. A manager coordinates all the activities of the members of the nursing staff in delivering nursing care and has personnel, policy, and budgetary responsibilities for a specific nursing unit or agency. An educator explains concepts and facts about health, demonstrates procedures such as self-care activities, reinforces learning or client behavior, and evaluates the client's progress in learning.

The nurse is caring for a client with end-stage lung disease. The client wants to go home on oxygen therapy and be comfortable. The family wants the client to undergo a new surgical procedure. The nurse explains the risk and benefits of the surgery to the family and discusses the client's wishes with the family. The nurse is acting as the client's: A) Manager B) Educator C) Advocate D) Caregiver

Answer: B Painless, pea-sized nodules should be checked by a health care provider. Testicular self-examination should be performed monthly and should be done after a bath or shower. The testes feel smooth, rubbery, and free of nodules.

The nurse is teaching a client how to perform a testicular self-examination. The nurse tells the client which of the following? A) "The testes are normally round, moveable, and have a lumpy consistency." B) "Contact your health care provider if you feel a painless pea-sized nodule." C) "The best time to do a testicular self-examination is before your bath or shower." D) "Perform a testicular self-examination weekly to detect signs of testicular cancer."

Collecting all available information about the situation

The nurse is working with parents of a seriously ill newborn. Surgery has been proposed for the infant, but the chances of success are unclear. In helping the parents resolve this ethical conflict, the nurse knows that the first step is

Answer: D Cheyne-Stokes respiration is an irregular respiratory rate and depth with alternating periods of apnea and hyperventilation; it begins with slow breaths and climaxes in apnea before respiration resumes.

The nurse observes that a client's breathing pattern represents Cheyne-Stokes respiration. Which statement best describes the Cheyne-Stokes pattern? A) Respirations cease for several seconds. B) Respirations are abnormally shallow for two to three breaths followed by irregular periods of apnea. C) Respirations are labored, with an increase in depth and rate (more than 20 breaths per minute); the condition occurs normally during exercise. D) Respiration rate and depth are irregular, with alternating periods of apnea and hyperventilation; the cycle begins with slow breaths and climaxes in apnea.

Answer: D The code of ethics is the philosophical ideals of right and wrong that define the principles the nurse will use to provide care to clients. A code of ethics does not ensure identical care to all clients (which would not be acceptable). The nursing code of ethics does not protect clients from harm or improve self-health care.

The nurse practices nursing in conformity with the code of ethics for professional registered nurses. This code: A) Improves self-health care B) Protects the client from harm C) Ensures identical care to all clients D) Defines the principles by which nurses' provide care to their clients

Answer: B Publication of research results provides other nurses with the scientific background of the study before they apply its findings in practice. Study subjects and setting should be similar to duplicate a study. Nurses should not change from accepted to unproven ways of providing care without careful research and collaboration with colleagues. Experimenting with new nursing measures is inappropriate and may place a client at risk.

The nurse researcher who gains new knowledge regarding a procedure can most effectively share the information with the nursing profession by: A) Duplicating the study using different clients in different settings B) Communicating the research findings in a professional journal C) Recruiting clients who are willing to demonstrate the new technique D) Asking individual nurses to report their experiences related to the new procedure

Answer: D Aortic, pulmonic, tricuspid, mitral areas are the sites for auscultation assessment of cardiac function. Auscultation of the carotid arteries is not the same as auscultation of the heart itself. The sternal region is not an appropriate site. There are no costal sites where heart sounds can be heard well. Anterior, posterior, and lateral are too vague.

The nurse should use which anatomical sites for the auscultatory assessment of cardiac function? A) Inner costal, outer costal, and sternal B) Aortic, carotid, coronary, and jugular C) Apical, lateral, anterior, and posterior D) Aortic, pulmonic, tricuspid, and mitral

Answer: C Pigmented skin lesions that are asymmetrical, have irregular borders, have variegated colors, and are larger than 6 mm in diameter are lesions that are suspect and should be reported to a medical provider.

The nurse teaches the client to inspect all skin surfaces and to report pigmented skin lesions that: A) Are symmetrical B) Are uniform in color C) Have irregular borders D) Are smaller than 6 mm in diameter

Answer: B Identified linkages of a nursing paradigm are the person, health, environment/situation, and nursing itself. Concepts, definitions, relationship, and assumptions are components of a theory. The individuals, groups, situations, and interests specific to nursing are potential subjects for middle-range theories. Description, explanation, prediction, and prescription of an interrelationship of nursing are purposes of nursing theory.

The nursing paradigm identifies four linkages of interest to the nursing profession. These four linkages are: A) Concepts, definitions, relationships, and assumptions B) The person, health, environment/situation, and nursing C) The individual, groups, situations, and interests specific to nursing D) Description, explanation, prediction, and prescription of an interrelationship of nursing

Answer: C An open system is defined as a system that interacts with the environment, exchanging information between the system and the environment.

The nursing process is an example of an open system. An open system: A) Is universal and dynamic B) Represents a relationship between two concepts C) Interacts with the environment by exchanging information D) Is a process through which information is returned to the system

Answer: B The question describes the nursing theory developed by Fay Abdellah and others. Rogers' theory considered the individual as an energy field existing within the universe. Henderson's theory defines nursing as "assisting the individual, sick, or well, in the performance of those activities that will contribute to health, recovery, or a peaceful death." Nightingale viewed nursing as providing fresh air, light, warmth, cleanliness, quiet, and adequate nutrition.

The nursing theory that emphasizes the delivery of nursing care for the whole person to meet the physical, emotional, intellectual, social, and spiritual needs of the client and family is: A) Rogers' theory B) Abdellah's theory C) Henderson's theory D) Nightingale's theory

Relationships

The philosophy sometimes called the code of ethics of care suggests that ethical dilemmas can best be solved by attention to

Answer: B Mary Mahoney, the first African American professional nurse, worked to bring respect to individuals regardless of race, color, background, or religion. Tubman assisted slaves to freedom during the Civil War. Hampton founded the Nurses Associated Alumnae of the United States and Canada, which later became the American Nurses Association (ANA). Nutting was instrumental in the affiliation of nursing education with universities.

The professional nurse responsible for increasing respect for the individual and awareness of cultural diversity was: A) Harriet Tubman B) Mary Mahoney C) Isabel Hampton D) Mary Adelaide Nutting

Answer: D The utilization review committee reviews admissions, diagnostic procedures, and treatments ordered by physicians. Review of the quality, quantity, and cost of care is more similar to the functions of a professional standards review organization. Review of reimbursement fees and appropriation of funds involves review of diagnosis-related groups. Reviewing the utilization of the payment mechanism is similar to capitation.

The purpose of a utilization review committee is to: A) Review quality, quantity, and cost of care B) Review the utilization of the payment mechanism C) Review reimbursement fees and appropriation of funds D) Review admissions, diagnostic tests, and treatments ordered by physicians

Answer: D The nurse begins with inspection and then follows with auscultation. It is important to perform auscultation before palpation and percussion, because palpation and percussion may alter the frequency and character of bowel sounds.

The techniques of physical assessment are inspection, palpation, percussion, and auscultation. The order in which these techniques are used is slightly different during abdominal examination than during examination of other body areas. The nurse should perform which two of the following first? A) Palpation and inspection B) Inspection and percussion C) Palpation and auscultation D) Inspection and auscultation

Answer: C Prescriptive theory addresses nursing interventions and predicts the consequence of a specific nursing intervention. Middle-range theories are limited in scope, less abstract than grand theories, address specific phenomena or concepts, and reflect practice. Descriptive theories describe phenomena, speculate as to why the phenomena occur, and describe the consequences of phenomena. Grand theories are broad and complex.

The type of theory that tests the validity and predictability of nursing interventions is: A) A grand theory B) A descriptive theory C) A prescriptive theory D) A middle-range theory

Answer: A Grand theories are described as broad and complex. Middle-range theories are limited in scope, less abstract, address specific phenomena or concepts, and reflect practice. Descriptive theories describe phenomena, speculate as to why the phenomena occur, and describe the consequences of phenomena. Prescriptive theories address nursing interventions and predict the consequence of a specific intervention.

Theories that are broad and complex are: A) Grand theories B) Descriptive theories C) Middle-range theories D) Prescriptive theories

Answer: C Theories will be tested to describe or predict client outcomes as nursing is addressed as a science and an art. Scientists will not make nursing decisions, and nursing will base client care on the practice of nursing science, which will be guided by multiple theories.

There is a contemporary move toward addressing nursing as a science or as evidenced-based practice. This suggests that: A) One theory will guide nursing practice. B) Scientists will make nursing decisions. C) Theories will be tested to describe or predict client outcomes. D) Nursing will base client care on the practice of other sciences.

Who is not able to start PPE

Those with liver problems and pregnant women

Answer: D Superficial lymph nodes are gently palpated using the pads of the index and middle fingers. The lymph nodes are small, and any other method would not be helpful.

To assess a client's superficial lymph nodes, the nurse: A) Deeply palpates using the entire hand B) Deeply palpates using a bimanual technique C) Lightly palpates using a bimanual technique D) Gently palpates using the pads of the index and middle fingers

Answer: D The dorsalis pedis pulse is felt on the top of the foot in line with the groove between the extensor tendons of the great toe and the first toe. The popliteal pulse is felt behind the knee. The posterior tibial pulse is felt in the groove behind the medial malleolus. Typically pulses are not palpated behind the lateral malleolus.

To assess the client's dorsalis pedis pulse, the nurse palpates: A) Behind the knee B) Over the lateral malleolus C) In the groove behind the medial malleolus D) Lateral to the extensor tendon of the great toe

Answer: A The sounds of lung fields on one side of the body are compared with the sounds of the same fields on the opposite side of the body. The other answers would provide incorrect comparisons.

To auscultate the client's lung fields, the nurse uses a systematic pattern comparing: A) Side to side B) Top to bottom C) Anterior to posterior D) Interspace to interspace

Answer: C The nurse accurately assesses temperature by palpating the skin with the dorsum or back of the hand, because this area of the hand is more sensitive to temperature than is the base of the hands, the fingertips, or the palmar surface.

To correctly palpate the client's skin for temperature, the nurse uses which of the following? A) Base of the hands B) Fingertips of the hands C) Dorsal surface of the hands D) Palmar surface of the hands

Answer: C Interdisciplinary theories provide a systematic view of a phenomenon. Developmental theories, health and wellness theories, and systems theories are examples of other types of theories.

To practice in today's health care environment, nurses need a strong scientific knowledge base in nursing and other disciplines, such as the physical, social, and behavioral sciences. This relates to which of the following? A) Systems theories B) Developmental theories C) Interdisciplinary theories D) Health and wellness model

RULE: TRANSFERRING ADULT TO PEDIATRIC WARDS

Transfer the adult patient with a condition or treatment similar to that which is seen in the pediatric population COPD is like CF (cystic fibrosis) = both need chest physiotherapy Pneumonia is pneumonia

True

True or False - The dominant value in American society of individual autonomy and self-determination may be in direct conflict with diverse groups.

False

True or False - The nurse practice acts are an example of civil law.

Answer: D To assess skin turgor, a fold of skin on the back of the forearm or sternal area is grasped with the fingertips and released. When turgor is good the skin lifts easily and snaps back immediately. The skin stays pinched when turgor is poor. The hands and neck are not the best places to test turgor, because the skin is normally loose and thin in those areas.

Turgor is the skin's elasticity, which can be diminished by edema or dehydration. Which is the best place for the nurse to assess skin turgor in the older adult? A) Side of the neck B) Back of the hand C) Palm of the hand D) Over the sternal area

Who needs Standard and Contact precautions (SC)

Typhus Psittacosis Meliodosis Shigellosis Brucellosis

Answer: A Although early morning temperatures are routinely low, the best practice is for the nurse to check the client's previous temperatures. Clients may routinely have a low temperature. Depending on the client's temperature history, the nurse may retake the temperature with another thermometer to check for a malfunction. If everything seems satisfactory, the nurse should chart the temperature and check the client for signs of hypothermia.

Using an oral electronic thermometer, the nurse checks the early morning temperature of a client. The client's temperature is 36.1° C (97° F). The client's remaining vital signs are in the normally acceptable range. What should the nurse do next? A) Check the client's temperature history. B) Document the results; temperature is normal. C) Recheck the temperature every 15 minutes until it is normal. D) Get another thermometer; the temperature is obviously an error.

The nurse asks the client's spouse, "Mrs. Smith, your husband told me that for the past week he has not been eating the meals you prepare. Do you agree?" This is an example of __________________ of assessment data.

Validation

Answer: D Vulnerable population are defined as clients who are more likely to develop health problems as a result of excess risks, who have limits in access to health care services, or who are dependent on others for care.

Vulnerable populations of clients are those who are more likely to develop health problems as a result of: A) Chronic diseases, homelessness, and poverty B) Poverty and limits in access to health care services C) Lack of transportation, dependence on others for care, and homelessness D) Excess risks, limits in access to health care services, and dependence on others for care

How can you prevent Salmonella

Wash hands before eating, after handling pets wash fruit and through cook meat and eggs

What should the school teacher teach kids handleing pets

Wash their hands before and after handling pets

How can you stop the spread of c diff

Wash your hands w/ an antimicrobial soap for 15min before and after contact with pts and teach pt's family members to do the same place in isolation, use dedicated equipment

Answer: C The community has three components: structure or locale, people, and social systems. To develop a complete community assessment, the nurse must take a careful look at each of the three components to begin to identify needs for health policy, health programs, and health services.

What are the three elements included in a community assessment? A) Environment, families, and social systems B) People, neighborhoods, and social systems C) Structure or locale, people, and social systems D) Health care systems, geographic boundaries, and people

The sequencing of stages of grief may occur in order, they may be skipped, or they may reoccur

When helping a person through grief work, the nurse knows

Current health standards should determine the acceptability of cultural practices.

When providing care to clients with varied cultural backgrounds, it is imperative for the nurse to recognize that:

Bio-cultural needs

Which factor is least significant during assessment when gathering information about cultural practices?

Answer: A Standards of care describe the competency level of nursing care as described by the ANA. The Nurse Practice Act regulates the licensing and practice of nursing; it describes the scope of practice. Accreditation allows the facility, school, or hospital to operate and be recognized in good standing according to standards set by peers. National council licensure is the standardized national examination that assess for a minimum knowledge base relevant to the client population that the nurse serves.

Which of the following assures clients that they will receive quality care from a competent nurse? A) Standards of care B) Nurse Practice Act C) Accreditation certification D) National council licensure

Answer: B Developmental theories discuss human growth from conception to death. The other options are incorrect.

Which theories describe an orderly process beginning with conception and continuing through death? A) Systems theories B) Developmental theories C) Interdisciplinary theories D) Stress and adaptation theories

Answer: D Florence Nightingale is the correct choice. Barton founded the Red Cross. Dix organized hospitals, nurses, and supply lines to support the troops of the Union Army. Wald opened the first community health service for the poor.

Who acted to decrease mortality by improving sanitation in the battlefields, which resulted in a decline in illness and infection? A) Dorothea Dix B) Lillian Wald C) Clara Barton D) Florence Nightingale

To be most effective when should postexposure prophylaxis (PPE) be started?

Within in 1 hour of exposure, but is still effective if given as late as 24 hours after exposure

Where would you see a rash w/ Rocky mountain spotted fever?

Wrist ankles Palms, soles Forearm *LOOK FOR TICK ANSWERS

After a bloodborne pathogen exposure what should be done later?

You should get tested for HIV HEP B,C @ 6 weeks, 3months, and 6 months

sitz bath

a bath used to apply heat to the pelvic area.

inflammation

a condition resulting from irritation in any body part, marked by pain, heat, redness, and swelling.

strabismus

a deviation of the eye; squint. (Convergent strabismus is called cross-eye; divergent strabismus is called exotropia or walleye. Other types include cyclotropia ,esotropia, hypertropia, and hypotropia.)

incentive spirometer

a device used to force the client to concentrate on inspiration and promote full inflation of the lungs, while providing immediate feedback; used particularly after surgery and in lung disorders.

chronic disease

a disease of long duration that generally manifests itself in an individual as recurring problems that tend to worsen in severity over time.

secondary disease

a disease that directly results from or depends on another disease.

primary disease

a disease that occurs independently, not related to another disease.

risk factor

a factor that increases a person's likelihood of developing a certain disease.

egg crate mattress

a foam pad, shaped like an egg carton, which is used on top of a regular bed mattress to provide comfort and to pre-vent pressure areas.

embolus

a foreign substance, blood clot, fat globule, piece of tissue, or air bubble carried in a blood vessel, which partially or completely obstructs blood flow (embolism; pl. emboli)

Hemoccult

a test for occult (hidden)blood in stool or body secretions.

Homans' sign

a test for thrombophlebitis in which pain occurs behind the knee when the foot is hyper flexed upward (dorsiflexion).

endoscope

a tube-shaped, lighted device used to visualize or operate on hollow organs or within body cavities. Specialized endoscopes include the gastroscope, broncho-scope, and proctoscope. (Process of visualization using this tool is called endoscopy.)

thrill

a vibrating sensation along an artery

emaciation

a wasting away of the flesh, causing extreme leanness, starvation. (adj: emaciated).

wheeze

a whistling respiratory sound, typical of asthma.

macule

a flat discolored spot on the skin (also, macule); a dense scar of the cornea that can be seen without optical aids.

Kardex

a flip file with card slots or a notebook for each client on a unit or nursing care team; a system for recording background information and care related to a client's treatment.

cognitive function

ability to think and reason.

diarrhea

abnormal frequency and fluidity of discharge from the bowels

neoplastic

abnormal growth of new tissue

herniation

abnormal protrusion of an organ or tissue through the structure usually containing it, as an inguinal hernia or hiatal hernia; rupture; condition is called herniation.

hypoxia

abnormal reduction of oxygen in the tissues.

contracture

abnormal shortening of muscles with resultant deformity.

bruits

abnormal swishing sounds heard over organs, glands or arteries

bradycardia

abnormally slow heart; slow pulse

pallor

absence of skin pigment; paleness.

fecal impaction

accumulation of hardened stool in the rectum.

nociceptive pain

acute pain; a pain sensation that results abruptly

acute pain

acute pain; a pain sensation that results abruptly.

accommodation

adjustment, as the accommodation of the lens of the eye.

postoperative

after surgery

objective data

all measurable and observable pieces of information about a client and his or her overall state of health

antiembolism stockings

also called TED socks; elastic stockings that cover the foot (not the toes) and the leg, up to the knee or mid-thigh.

residual urine

amount of urine that remains in the bladder after voiding at least once.

calculi

an abnormal concretion usually composed of mineral salts, occurring in the hollow body organs; a "stone," as a calculus in the kidney (pl: calculi); deposit on the teeth (tartar).

alias

an assigned name under which certain clients are admitted to (and records kept in) a healthcare facility in order to maintain anonymity.

short-term goal

an expected outcome or goal that a client can reasonably meet in a matter of hours or days.

short-term objective

an expected outcome or goal that a client can reasonably meet in a matter of hours or days.

sequela

an illness or injury that follows as a direct result of a previous condition or event.

enema

an injection of fluid or medication into the rectum, usually to induce evacuation of the bowel

urinometer

an instrument that deter-mines urine's specific gravity; also called urometer, hydrometer.

personal space

an invisible, mutually understood area or zone around a per-son that is considered inappropriate for strangers to violate (varies between cultures). If a person invades another's personal space (comes too close), it may cause discomfort. Much nursing care must occur within the client's personal space.

long-term goal

an outcome or goal that a client hopes to achieve but may require an extended amount of time to do so

long-term objective

an outcome or goal that a client hopes to achieve but may require an extended amount of time to do so.

complication

an unexpected event in a disease's course that delays a person's recovery.

halitosis

bad breath

base of support

balance or stability provided by the feet and their positioning

base of support

balance or stability provided by the feet and their positioning.

tepid sponge bath

bath with water below body temperature, 70 to 85º F, used to reduce fever

perineal care

bathing genitals and surrounding area

unoccupied bed

bed that is empty at the time it is made up.

wound sinus

canal or passage leading to an abscess.

inspection

careful, close, and detailed visual examination of a body part.

elective

case in which the client's condition is not life-threatening and may choose whether or not to have surgery; also called optional surgery

venous access lock

catheter used to maintain an open route to a client's venous system to give fluids and/or medications.

etiology

cause

apnea

cessation of breathing

neuropathic pain

chronic pain or discomfort that continues for six months or longer and interferes with normal functioning.

circumduction

circular movement of a limb or the eye.

acuity

clearness; or a disorder's level of severity; minimum level or need for healthcare services that must be met for a client to be admitted to an acute care facility.

anesthesia

complete or partial loss of sensation.

anuria

complete suppression of urine secretion in the kidney.

bradypnea

condition in which breaths are abnormally slow and fall below ten per minute.

conscious sedation

condition in which internal sedative medications are used alone or in conjunction with local anesthetics and the client has a depressed level of consciousness but is still able to breathe and respond to verbal stimuli.

purulent

consisting of or secreting pus.

serous

containing clear fluid; drainage made up of serum

footdrop

contracture deformity that prevents the client from putting the heel on the floor; results from improper positioning or anterior leg muscle paralysis; abnormal plantar flexion of the foot.

footdrop

contracture deformity that prevents the client from putting the heel on the floor; results from improper positioning or anterior leg muscle paralysis; abnormal plantar flexion of foot.

hypothermia blanket

cooling blanket; also called hypothermia blanket

pyorrhea

copious discharge of pus

urgency

desire or sensation of needing to void immediately.

oliguria

deficient urinary secretion or infrequent urination

orthopneic position

difficult breathing, relieved by sitting or standing erect; orthopneic position: sitting and leaning forward, to facilitate breathing.

constipation

difficult or infrequent and hardened bowel movements.

dysuria

difficult or painful urination or voiding.

dysphasia

difficulty in understanding or expressing language.

gravital plane

direction of gravitation pull; an imaginary vertical line through the top of the head, center of gravity, and base of support.

line of gravity

direction of gravitation pull; an imaginary vertical line through the top of the head, center of gravity, and base of support.

defecation

discharge of solid waste matter (feces) from the intestines.

acute disease

disease or illness that develops suddenly and runs its course in days or weeks; illnesses that interfere with the continuum for a short period of time.

local anesthesia

disruption of sensation to a specific body area without causing unconsciousness; caused by infiltration or topical application of anesthetic, usually to a small area; not general.

nursing progress notes(nurses' notes):

documentation by nurses of care given and observations made; charting data input

Montgomery straps

easily removable straps that stay in place to facilitate dressing removal

projectile vomiting

emesis expelled with great force.

goal-oriented

establishment of objectives or specific desired outcomes early in the nursing process.

urinalysis

examination of urine

lithotomy

examination position in which the client is lying on his or her back with the feet in stirrups.

Fowler's position

examination position in which the client is lying on his or her back with the head elevated.

Sims' position

examination position in which the client is lying on his or her left side with right knee flexed.

hemorrhage

excessive bleeding(internal or external); escape of blood from non-intact blood vessels.

nocturia

excessive voiding (urination) during the night.

Kegel exercises

exercises designed to increase sphincter tone by tightening, holding, and releasing the muscles of the pelvic floor and sphincter, used to improve incontinence.

traction

exertion of a pulling force; an apparatus attached to the client to maintain stability of a joint or aligned fracture or to exert a pulling force elsewhere, as in the lower back, to relieve pressure.

trial and error problem-solving

experimental problem solving that tests ideas to decide which methods work and which do not.

auscultation

externally listening to sounds from within the body to determine abnormal conditions, as auscultation of blood pressure with a stethoscope

auscultation

externally listening to sounds from within the body to determine abnormal conditions, as auscultation of blood pressure with a stethoscope.

friable

fagile; easily broken

body cue

feelings experienced in response to body rhythms, self-monitoring

exudate

fluid cells or other substances coming through pores or breaks in the skin

crackles

fluid in the bronchioles and alveoli heard as crackling or bubbling on inspiration

client-oriented

focused on meeting individualized needs.

thrombophlebitis

formation of a blood clot in a vein, with inflammation

thrombophlebitis

formation of a blood clot in a vein, with inflammation.

suppuration

formation or discharge of pus (adj: suppurative).

sordes

foul, dark matter that collects around the teeth and lips in low grade fever

friable

fragile; easily broken

symptom

functional evidence of a disease or condition that a client perceives subjectively (as opposed to signs, which the examiner or others perceive).

flatus

gas in the intestines or stomach; gas expelled through the anus.

verbal communication

giving information, news, or messages by speaking or writing.

communication

giving, receiving, and interpreting information (maybe verbal or nonverbal).

nursing care plan

guidelines used by healthcare facilities to plan the care for clients.

isometric

having the same length or dimensions, as isometric exercises(pushing against stable resistance);also called muscle setting.

occult

hidden

borborygmi

high pitched loud rushing bowel sounds

sibilant

high pitched musical wheezes

trapeze

horizontal bar suspended above and attached to the bed, which is used to pull up to a sitting position or to lift the shoulders and hips off the bed.

body language

impressions one conveys through body movements and posture, eye contact, and other non-verbal means.

planning

in nursing process, developing goals to prevent, reduce, or eliminate problems and identifying nursing interventions that will assist in meeting these goals.

planning

in nursing process, developing goals to prevent, reduce, or eliminate problems and identifying nursing interventions that will assist in meeting these goals..

evaluation

in nursing process, measuring the effectiveness of the other steps.

implementation

in nursing process, the carrying out of nursing care plans; also called interventions.

potential needs

in the nursing process, needs which may occur; identified as at risk for...

incontinence

inability to control urination or defecation (adj: incontinent).

urinary retention

inability to empty the bladder of urine.

pediculosis

infested with lice

subjective data

information that consists of the client's opinions and feeling about what is happening, conveyed to the nurse either directly or through body language

hypoxemia

interference with blood oxygenation.

regional anesthesia

interruption of sensory nerve conductivity to specific area of the body (includes conduction block, field block, nerve block).

infection

invasion of microorganisms

enuresis

involuntary urine discharge, usually occurring during sleep; bedwetting.

cystitis

inflammation of any bladder(most often refers to urinary bladder)

pruritus

itching

scoliosis

lateral curvature of the normally straight, vertical line of the spine, sometimes is S-shaped ("curvature of the spine").

LOC

level of consciousness

nits

lice eggs

auscultation

listening to sounds produced by the body

eye contact

looking another person in the eye, as in "making eye contact."

hypothermia

low body temperature; also a syndrome (accidental hypothermia), caused by exposure to cold, which may be fatal. Hypothermia may also be induced for therapeutic purposes such as surgery, or pathologic as a result of faulty thermoregulation (temperature control)

sonorous

low pitched coarse gurgling or snoring

pain threshold

lowest intensity of a stimulus that causes a subject to recognize pain.

pneumonia

lung inflammation, with consolidation and drainage

supine

lying on back

assessment

making an evaluation or appraisal of a patient's condition

gait

manner or style of walking

Droplet precautions

mask will 3 feet

exudate

material that escapes from blood vessels and is deposited in tissues or on tissue surfaces; usually contains protein substances.

flotation mattress

mattress or pad filled with a gel-type material which supports the body in a way to pro-vide comfort and avoid creating pressure points, thereby helping to prevent skin breakdown.

expected outcome

measurable behavior that indicates whether a person has achieved the expected benefit of nursing care

vital signs

measurements of temperature, pulse, respiration, and blood pressure.

peripheral neurovascular assessment

method for evaluating the status of an extremity in a bandage or case

logroll turn

method of turning a client that keeps the body in straight alignment, used for clients with injuries to the back and/or spinal cord.

granulation tissue

new tissue that forms when old destroyed tissue is sloughed off.

nociception

normal pain transmission

sign

objective evidence of disease that another person can note (as opposed to symptom, which only the client can describe).

intraoperative

occurring during a surgical operation.

crackle

on auscultation, an abnormal discontinuous non-musical respiratory sound heard on inspiration; formerly called rale.

dehiscence

opening or separation of the surgical incision.

gurney

our-wheeled cart; also called gurney, wheeled stretcher. A litter scale is used to weigh clients who cannot stand.

aquathermia pad

pad which produces a dry heat by the use of temperature-controlled water flowing through a waterproof shell.

trochanter roll

padding placed onsides of legs and feet of a client in bed, to prevent abnormal outward rotation and related sequela.

chronic pain

pain that lasts more than six months; neuropathic pain.

symptoms

pain, nausea, any sensation described by the patient

paraplegia

paralysis of the legs and sometimes the lower part of the body; a person with this condition is called a paraplegic.

hemiplegia

paralysis on one side of the body.

melena

passage of dark-colored stools containing partially or fully digested blood; also used to mean abnormal blood in the stool or vomitus.

micturition

passage of urine from the urinary bladder; also called voiding, urinating.

ulcer

pen sore on an external or internal body surface that causes gradual disintegration of tissues, often an ulcer of the stomach (pepticulcer) or a pressure sore (decubitus ulcer).

Who shouldn't get the measles vacc

people w/ egg allergies pregnant illness or fever

dullness

percussion over a dense organ produces a thudlike sound

pain tolerance

point at which a per-son can no longer tolerate pain.

prone

positioning a client so that he or she is lying on the stomach

prone

positioning a client so that he or she is lying on the stomach.

dangling

positioning of a client so that he or she is sitting on the edge of the bed with legs down and feet supported by a footstool or the floor. This is an exercise in preparation for sitting in a chair and/or walking.

scientific problem-solving

precise method of investigating problems and arriving at solutions

prioritization

prioritizing: in the nursing process, following specific steps to determine the client's most important needs.

What pt's need Droplet precautions?

private room gloves gown MRS.WEE M - Multidrug resistant organism R - Respiratory infection - RSV child wears mask S - Skin infections (see below "VCHIPS") W - Wound infections E - Enteric infections - clostridium defficile E - Eye infections VCHIPS V - Varicella zoster C - Cutaneous diptheria H - Herpes simplex I - Impetigo P - Pediculosis S - Scabies, Staphylococcus

collaborative problem

problem in which nurses work with physicians or other healthcare providers.

rotation

process of turning about an axis, as rotation of the hand

rotation

process of turning about an axis, as rotation of the hand.

prognosis

projected client outcome.

inflammation

protective response of body tissues to irritation, injury or invasion by germs

carotid pulse

pulse felt on either side of the neck, over the carotid artery.

apical pulse

pulse normally heard at the heart's apex, which usually gives the most accurate assessment of pulse rate.

venipuncture

puncture of a vein, usually with a needle. May be used to obtain a blood specimen or to start an intravenous infusion (IV).

inspection

purposeful observation

purulent

pus

closed-ended question

questions that can usually be answered by one word, such as "yes" or "no;" also called close-ended questions.

open-ended question

questions used in therapeutic communication and interviews that promote in-depth answers and encourage clients to talk about themselves and their concerns.

biopsy

removal of a sample of body tissue or fluid for diagnostic examination, usually microscopic; most often used to detect the presence of cancer.

drainage

removal of fluids from a body cavity or wound.

protective device

same as client reminder device

litter

same as gurney

keloid

scar or scar tissue

smegma

sebaceous gland secretion that may collect under foreskin of penis in an uncircumcised male.

signs

seen heard measured or felt

renal colic

severe, penetrating lower back pain, caused by a stone becoming lodged in the ureter

lateral

side-lying

objective data

signs seen by the nurse

erythema

skin redness produced by capillary congestion, as may follow a tuberculin test; bright red color associated with capillary dilation, can indicate fever or infection.

turgor

skin resiliency and plumpness; also called skin turgor.

vesicle

small sac containing liquid; small blister.

papule

small, solid, circumscribed skin elevation, less than 0.5-1.0 cm in diameter.

maceration

softening of a solid due to soaking, until connective tissue fibers are dissolved, such as maceration of the skin under a cast or bandage

wheezes

sounds produced by movement of air through narrowed passages in the traceobronchial tubes

cancer pain

specific type of pain identified by IASP, caused by a malignancy; often intractable and severe; usually chronic

expectorate

spitting out and coughing up mucus or other fluid from the lungs and the throat.

medical diagnosis

statement formulated by a primary healthcare provider that identifies the disease a person is believed to have, which provides a basis for prognosis and treatment decisions.

vomitus

stomach contents expelled by vomiting or emisis

urinary suppression

stopping or inhibition of urination. Suppression of secretion urine is not formed. Suppression of excretion urine is not expelled

striae

stretch marks

gait belt

sturdy webbed belt used by the nurse to help provide support to the weak or unsteady person.

transfer belt

sturdy webbed belt used by the nurse to help provide support to the weak or unsteady person.

edema

swelling

What are the characteristics of Mumps

swelling and tenderness of one or more salivary glands

distention

swelling or fullness, as in urinary distention.

subjective data

symptoms perceived by the patient

nursing assessment

systematic and continuous collection and analysis of information about the client

nursing process

systematic method in which the nurse and client work together to plan and carry out effective nursing care. (The steps include assessment, nursing diagnosis, planning, implementation, and evaluation.)

infection

the invasion and multiplication of infective agents in body tissues with a resultant reaction(illness or injury) to their presence and/or their toxins.

contralateral

the opposite side

contralateral

the opposite side.

perioperative

the period surrounding surgery; includes the preoperative, intraoperative, and postoperative periods

evisceration

the protrusion of the intestines through an abdominal wound; removal of the internal body contents.

mitered (corners)

the type of beveled corners used when making a hospital bed.

axillary

the underarms

proxemics

the use of space in relationship to communication.

suture

thread used to hold an incision together while it heals; also called stitches.

necrosis

tissue death

voiding

to cast out wastes, as to urinate, micturate.

dehumanization

to make a person/client feel like an object, to remove one's dignity.

slough

to shed; to cast off (noun: slough—a mass of dead tissue).

guaiac

tool examination for blood; also known as Hemoccult.

urinary catheter

tube inserted into the bladder through the urethra to remove urine.

inversion

turning inside out; reversing.

eversion

turning inside out; turning outward

pronation

turning the hand so that the palm faces downward or backward.

thrombolytic

type of medication designed to dissolve a clot and clear a blocked blood vessel.

nodule

type of skin lesion appearing as a small knot or protuberance

Kerlix

type of stretchy gauze used to hold dressings in place.

palpation

use of hands and sense of touch to gather information

health interview

way of soliciting information from the client; may also be called a nursing history.

serosanguineous

fluid drainage com-posed of serum and blood.

35-65 yrs

(Middle Adulthood) Generativity V stagnation

1 year to 3 yr

(Toddler) Autonomy V Shame & doubt

birth to 1 yr

(infancy) Trust V Mistrust

6-12yrs

(school-age) industry V inferiority

47. Nursing-initiated interventions are: A. Determined by state nursing practice acts B. Supervised by the entire health care team C. Made in concert with the plan of care initiated by the physician D. Developed after evaluating the interventions for the recent medical diagnoses

A

57. A client with a long standing history of diabetes mellitus is voicing concerns about kidney disease. The client asks the nurse where urine is formed in the kidney. The nurse response is the: A. Glomulerus B. Kidney C. Nephron D. Ureter

A

62. The nursing assistant asks you the difference between a wound that heals by primary or secondary intention. You will reply that a wound A. Are approximated B. Migrate across the incision C. Appear slightly pink D. Slightly overlap each other

A

7. Helping relationship is the foundation of clinical nursing practice. Contracts for therapeutic helping relationship are formed during the: A. Orientation Stage B. Working stage C. Termination D. Pre-interaction stage * definitely look at all these stages when you're studying

A

8. Information regarding a client's health status may not be released to non-health care team members because: A. Legal and ethical obligations require health care providers to keep information strictly confidential B. Regulations require health care institutions document evidence of physical and emotional well being C. Reimbursement issues relating to patient care and procedures may be of concern D. A fragmentation of nursing and medical care procedures may be identified

A

A client is receiving health care from a health care provider who is a salaried employee. Which model is being followed by the managed care organization (MCO) to which the client belongs? (Select all that apply.) A) Staff model B) Group model C) Network model D) Independent practice association

A

Nursing practice in the twenty-first century is an art and science that is centered on: A) The client B) The nursing process C) Cultural diversity D) The health care facility

A

When action is taken on one's prejudices: A) Discrimination occurs. B) Effective intercultural communication develops. C) Delivery of culturally congruent care is ensured. D) Sufficient comparative knowledge of diverse groups is obtained.

A

Which factors are least significant during assessment when gathering information about cultural practices? A) Biocultural needs B) Language, timing C) Touch and eye contact D) Pain perception and pain management expectations

A

Which of the following is not a normal physiological change associated with aging? A) Osteoporosis B) Decreased cardiac output C) Reduced ability to see in darkness D) Smooth, brown, irregularly shaped spots on the backs of the hands and forearms

A

Older adults are at increased risk for drug toxicity because they: A) Have reduced kidney functioning B) Have increased serum albumin levels C) Often take their medications incorrectly D) Have decreased adipose tissue to store lipid-soluble drugs

A A decreased number of functioning nephrons and decreased glomerular filtration rate in older adults place them at greater risk for drug toxicity for drugs excreted through the renal system. The biological half-life is extended, and drugs take longer to be filtered from the body thus, the risk of adverse reactions is increased. The older adult has reduced (not increased) serum albumin levels, so that protein-bound drugs must compete for protein-binding sites. If the serum albumin level is low, the client is at greater risk for toxicity even when blood levels of the drug are normal or low. The older adult has increased adipose tissue compared to lean body mass. Drugs stored in adipose tissue (lipid-soluble drugs) have increased tissue concentrations and accumulate and remain in the body longer. Option 3 is a false statement generalizing the behavior of older adults.

Which of the following persons can legally give consent to a procedure? A) An appointed guardian B) An unemancipated minor C) Only individuals who are 21 years of age or older D) An advocate for a child

A A legal guardian is an individual who has been appointed by a court and has full legal rights to make choices regarding care for another individual. An emancipated minor can make care choices, but one who is not emancipated must seek permission from his or her guardian or parent for procedures. A client who is at least 18 years of age, not 21 years, can give permission for procedures as long as the client is competent. An advocate does not have a legal right to choose care management. An advocate speaks for a child or an adult when the person cannot speak for himself or herself but does not have the legal right to make choices about care or to give consent.

Recent research provided evidence that a professional nursing staff affects health care financing. These results indicated that the positive benefit of a professional nursing staff is: A) Decreased length of stay B) Decreased rate of readmission C) Increased rate of nosocomial infections D) Decreased need to hire ancillary personnel

A A positive benefit of a professional nursing staff is a decreased length of stay. The diagnosis-related group has greater influence on the rate of readmission. The ancillary personnel need to remain so that registered nurses can spend the necessary time to assess and manage clients. Nosocomial infections decrease with a professional nursing staff

To enhance effectiveness in teaching the older adult, the nurse should: A) Speak in clear, low-pitched tones. B) Realize that older adults are less able to learn. C) Present an overview of several ideas at one time. D) Present abstract material rather than concrete material.

A An effective teaching strategy is to keep the voice pitch low and to speak at a moderate rate and volume. Older adults can hear low-frequency sounds better than high-frequency sounds. Option 2 is a false stereotype of the older adult. Although the process of learning may be affected by age-related changes in vision or hearing or by reduced energy and endurance, older adults are lifelong learners. The nurse should present concrete rather than abstract material to facilitate learning by older adults. Ideas should be presented one at a time to enhance learning

Which of the following statements is correct? A) Consent for medical treatment can be given by a minor with a sexually transmitted disease (STD). B) A second-trimester abortion can be performed without state involvement. C) Student nurses cannot be sued for malpractice while in a nursing clinical class. D) Nurses who get sick and leave during a shift are not abandoning their clients if they call their supervisor and leave a message about their emergency illness.

A Anyone, at any age, can be treated for an STD without parental permission. The client is "advised" to contact sexual partners but is not required to give their names. A first-trimester abortion can be given without state regulation because of the low risks of mortality. Student nurses are liable for their performance based on their level of knowledge at the time of providing care. Abandonment rules do apply if the nurse leaves without waiting for a replacement to arrive and/or talking directly to the supervisor.

An APN is the most independently functioning of all professional nurses. All of the following are examples of a clinically focused APN except: A) Care provider B) Case manager C) Nurse specialist D) Nurse practitioner

A Care provider is a staff position, a nurse who provides direct care. The nurse specialist has clinical expertise in a specific area. The nurse practitioner has advanced training in assessment and pharmacology and is able to provide health care in specific settings. The case manager has additional experience and is able to coordinate activities of other members of the health care team.

Which of the following is an example of respite care? A) Day care B) Home care C) Nursing home D) Nurse extender

A Day care is an example of respite care because it allows the family to maintain normalcy while the client is under their care. A nursing home client receives 24-hour care in the facility. Home care is an intermittent service in which only certain tasks are performed. Nurse extenders may be hired to perform a specific task, such as bathing.

Which of the following professional organizations was created to address concerns of members in the nursing profession? A) NLN B) MSN C) PHA D) NIH

A National League for Nursing (NLN) is the correct answer. The master of science in nursing (MSN) degree is earned through advanced educational preparation in nursing. Public Health Administration (PHA) is concerned with areas of public health. The National Institutes of Health (NIH) addresses health on a national level.

Which of the following is an example of transpersonal communication? A) Prayer B) Negotiation C) Active listening D) Positive self-talk

A Prayer is one form of transpersonal communication, in which interaction takes place in one's spiritual domain. Negotiation is often seen in the one-to-one interaction of interpersonal communication. Positive self-talk is a form of intrapersonal communication that can be used to improve one's self-esteem. Active listening is an important principle for effective communication in small groups.

General health promotion and illness prevention measures the nurse may recommend to older adults include: A) Exercising regularly B) Taking medications in the morning C) Receiving influenza vaccine every 6 to 8 years D) Undergoing immunization for pneumococcal pneumonia annually

A Regular exercise is a general preventative measure the nurse may recommend. Others include weight reduction if the client is overweight, management of hypertension, smoking cessation, and updating of immunizations. Annual influenza immunization of all older adults is strongly recommended. Approximately 95% of deaths in the United States from influenza occur among adults aged 65 and older. Pneumococcal pneumonia vaccine is given only once, although some authorities recommend revaccination 6 to 8 years after the initial vaccination. This vaccine is recommended for all adults over age 65. Older adults should take medications as p

Nurses are bound by a variety of laws. Which of the following descriptions of types of law is correct? A) Statutory law is created by elected legislatures, such as the state legislature that defines the Nurse Practice Act (NPA). B) Regulatory law provides for prevention of harm to the public and punishment when those laws are broken. C) Common law protects the rights of the individual within society to fair and equal treatment. D) Criminal law creates boards that pass rules and regulations to control society.

A Statutory law is created by legislatures. These bodies enact statutes such as the NPA, which defines the role of the nurse and expectations of the nurse's performance of his or her duties and explains what is contraindicated as guidelines for the breach of those regulations. Regulatory law or administrative law is created by an administrative body such as the Board of Nursing, which passes rules and regulations. Common law is created by judicial decisions in court based on individual cases that are decided. Criminal law prevents harm to society and provides punishment for crimes.

A nurse notes that the health care unit keeps a listing of clients' names at the front desk in clear view so that health care providers can more efficiently locate clients. The nurse knows that this action is a violation of which act? A) Health Insurance Portability and Accountability Act B) Emergency Medical Treatment and Active Labor Act C) Patient Self-Determination Act D) Mental Health Parity Act

A The Health Insurance Portability and Accountability Act provides for client privacy and confidentiality. The Emergency Medical Treatment and Active Labor Act ensures that any individual may receive appropriate emergency care. The Patient Self-Determination Act allows clients to determine their course of care. The Mental Health Parity Act forbids health plans from placing lifetime or annual limits on mental health coverage that are less generous that those placed on medical or surgical benefits.

To facilitate communication with an older adult who is hard of hearing the nurse should: A) Face the client and maintain eye contact. B) Use lengthy explanations to ensure that the message is made clear. C) Cover several topics at one time to be most efficient in communication. D) Help the client by anticipating what he or she is going to say and finishing the client's sentences for him or her.

A The nurse should get the client's attention before speaking and face the client so the client can see the nurse's mouth. The nurse should speak slowly and clearly while maintaining eye contact. Words should be supplemented with visual gestures. The nurse should suppress the desire to finish the client's sentences. The nurse should allow the client to make errors. Short sentences with simple words should be used. The nurse should stick to one topic at a time.

To meet the psychosocial needs of the older adult, the nurse may: A) Use appropriate therapeutic touch. B) Provide large-print reading material and bright light. C) Use blue, green, and pastel shades to help create landmarks. D) Increase the use of salt and sugar to compensate for a diminished sense of taste.

A Touch is a therapeutic tool that nurses can use to help comfort the older adult. Appropriate touch can help combat feelings of social isolation and rejection while supporting the older adult's self-concept and self-esteem. A decreased sense of taste is a physiological change, not a psychosocial need. Older adults have altered color perception and have increased difficulty discriminating between blue, green, and pastel shades. Although large-print material and more ambient light may be helpful, older adults have increased sensitivity to the effects of glare. These interventions do not meet a psychosocial need.

Which of the following should the nurse do when planning nursing care for a client with a different cultural background? A) Identify how these cultural variables affect the health problem. B) Speak slowly and show pictures to make sure the client always understands. C) Allow the family to provide care during the hospital stay so that all rituals or customs are carried out. D) Explain how the client must adapt to hospital routines to be effectively cared for while in the hospital.

A Without assessment and identification of the client's cultural needs, the nurse cannot begin to understand how these might influence the health problem or health care management. Although the family can assist if they desire to do so, the nurse is legally responsible for giving care based on current assessments, for documenting problems, and for followin

Remove all tubes and equipment (unless organ donation is to take place), clean the body, and position appropriately.

A client who had a "Do Not Resuscitate" order passed away. After verifying there is no pulse or respirations, the nurse should next:

What should you have the patient sign after being stuck by a needle?

A consent form to be signed for Hep B, C and HIV

The choices involved do not appear to be clearly right or wrong.

A health care issue often becomes an ethical dilemma because

What techniques encourage a client to tell his or her full story? (Select all that apply.) A) Active listening B) Back channeling C) Use of open-ended questions D) Use of closed-ended questions

A) Active listening B) Back channeling C) Use of open-ended questions Options 1, 2, and 3 encourage clients to tell their full stories. Closed-ended questions allow clients to answer with one or two words, which makes it more difficult to obtain all the information required for a full story. The other options give clients the opportunity to tell their stories and feel supported. Active listening helps them feel that they, and their stories, are important.

The nurse gathered the following assessment data. Which of these cues form a pattern? (Select all that apply.) A) Client is restless. B) Respirations are 24/min and irregular. C) Client states feeling short of breath. D) Fluid intake for 8 hours is 800 ml. E) Client has drainage from surgical wound. F) Client reports loss of appetite for over 2 weeks.

A) Client is restless. B) Respirations are 24/min and irregular. C) Client states feeling short of breath. The data in items 1, 2, and 3—rapid irregular breathing, complaints of shortness of breath, and restlessness—form a pattern indicating that the client may be experiencing hypoxia, because all are signs and symptoms characteristic of this condition. The other information, although important, is not related to hypoxia.

During the orientation phase of the helping relationship, the nurse might do which of the following? A) Discuss the cards and flowers in the room. B) Work together with the client to establish goals. C) Review the client's history to identify possible health concerns. D) Use therapeutic communication to manage the client's confusion.

A, B, C In the orientation phase of the helping relationship, the nurse and client meet and get to know each other. The nurse reviews the history to identify possible health concerns before meeting the client. The nurse and client also work together to solve problems and accomplish goals. Therapeutic communication skills are used during the working phase to facilitate successful interactions.

Contemporary nursing requires that the nurse possess knowledge and skills to carry out a variety of professional roles and responsibilities. Examples include which of the following? (Select all that apply.) A) Autonomy and accountability B) Advocacy C) Provision of bedside care D) Health promotion and illness prevention

A, B, C, & D

A client newly admitted to the hospital begins to have chest pain. Before calling the physician, the nurse should gather what additional data? (Select all that apply.) A) Pain intensity B) Location of pain C) Character of pain D) Radiation of pain E) Meaning of pain to the client F) Family history of myocardial infarctions

A, B, C, D, E

The nurse must follow standards of care to avoid potential litigation such as negligence suits. Which of the following describe a potential nursing malpractice situation? (Select all that apply.) A) Failure to question a health care provider about the appropriateness of a client order B) Failure to make a nursing diagnosis C) Failure to properly use medical equipment ordered for client monitoring D) Failure to follow the "six rights" of medication administration E) Failure to provide discharge instructions

A, B, C, D, E All of the answers represent possible nursing malpractice situations.

A nurse is planning a client's discharge from a subacute care unit to home. Education should be provided on which of the following topics? (Select all that apply.) A) Medication administration B) Stress reduction techniques with blood pressure assessment C) Circumstances in which the client should call the health care provider D) Hand-washing hygiene when assisting with transfer to the bathroom

A, C Clients being discharged home need education regarding how to take their medication and when to call their health care provider. There is not enough information here to determine if options 2 and 4 are appropriate, although hand hygiene after toileting is always important.

What would you question with a c diff pt

An order for and Anti Diarrhea med like Imodium

21. If a nurse decided to withhold a medication because it might further lower the client BP, the nurse will be practicing the principle of: A. Responsibility B. Accountability C. Competency D. Moral behavior

B

24. You are about to administer an oral medication and you question the dosage. You should: A. Administer the medication B. Notify the physician C. Withhold the medication D. Document the dosage appears incorrect

B

26. Nursing process is central to nursing practice. Nursing practice is: A. A theory B. Derived from a theory C. Not adaptable to all clients D. Generates knowledge for use in practice

B

27. A newly admitted client was found wandering the hallways for the past two nights. The most appropriate nursing intervention to prevent a fall for this client would include: A. Raise all four side rails when darkness falls B. Use an electronic bed monitoring device C. Place the client in a room close to the nursing station D. Use a loose-fitting vest-type jacket restraint

B

3. A client who needs nursing and rehabilitation following a stroke would most benefit from receiving care at a: A. Primary care center B. Restorative care setting C. Assisted living center D. Respite center

B

30. You are caring for a non-English speaking male client When preparing to assist him with personal hygiene, you should: A. Use soap and water on all types of skin B. Ensure that culture and ethnicity influence hygiene practices C. Shave facial hair to make the client more comfortable D. Know that all clients need to be bathed daily

B

38. A client suffers from sleep pattern disturbance. To promote adequate sleep, the most important nursing intervention is to: A. Administering a sleep aid B. Synchronizing the medication, treatment, and vital signs schedule C. Encouraging the client to exercise immediately before sleep D. Discussing with the client the benefits of beginning a long-term night time medication regimen

B

40. A client is admitted to the hospital with shortness of breath. As the nurse assesses this client, the nurse is using the process of: A. Evaluation B. Data collection C. Problem identification D. Testing hypothesis

B

44. A patient is suffering from shortness of breath. The correct goal statement would be written as: A. The client will be comfortable by the morning B. The client will breath unlabored at 14-18 breaths per minute by the end of the shift C. The client will not complain of breathing problems within the next 8 hours D. The client will have a RR of 14-18 breathes per min

B

51. A newly graduated nurse is assigned to care for a team consisting of herself and a certified nursing assistant. When delegating skills she needs to: A. Assign only bed-making skills and feeding tasks B. Assess the knowledge of the CNA C. Remind the staff member that she is working under the licences of the RN D. Allow the staff member to perform only skills that the RN is able to teach CNAs to perform

B

6. While admitting a client, during the initial interview a family member tells you " my mom really means that she does not understand her medical diagnosis." The communication form the family member is used: A. Focusing B. Clarifying C. Summarizing D. Paraphrasing

B

60. A newly admitted client states that he has recently had a change in medications and reports that stools are now dry and hard to pass. This type of bowel pattern is consistent with: A. Abnormal defecation B. Constipation C. Fecal impaction D. Fecal incontinence

B

61. To maintain normal elimination patterns in the hospitalized client, you should instruct the client to defecate 1 hour after meals because: A. The presence of food stimulates peristalsis B. Mass colonic peristalsis occurs at this time C. Irregularity helps to develop a habitual pattern D. Neglecting the urge to defecate can cause diarrhea

B

9. A nurse had just admitted a client with a medical diagnosis of CHF. When completing the admission paper work, the nurse needs to record: A. An interpretation of client behavior B. Objective data that are observed C. Lengthy entry using lay terms D. Abbreviations familiar to the nurse

B

A 46-year-old woman from Bosnia came to the United States 6 years ago. Although she did not celebrate Christmas when she lived in Bosnia, she celebrates Christmas with her family now. This woman has experienced assimilation into the culture of the United States because she: A) Chose to be bicultural B) Adapted to and adopted the American culture C) Had an extremely negative experience with the American culture D) Gave up part of her ethnic identity in favor of the American culture

B

Ethnocentrism is the root of: A) Cultural beliefs B) Biases and prejudices C) Meanings by which people make sense of their experiences D) Individualism and self-reliance in achieving and maintaining health

B

In the older adult taste buds atrophy and lose sensitivity and appetite may decrease. The older adult is therefore less able to discern: A) Spicy and bland foods B) Salty, sour, sweet, and bitter tastes C) Hot and cold food temperatures D) Moist and dry food preparations

B

The best explanation of what Title VI of the Civil Rights Act mandates is the freedom to: A) Pick any physician and insurance company despite one's income. B) Enjoy equal access to all health care regardless of race and religion. C) Receive free medical benefits as needed within the county of residence. D) Receive basic care under a sliding scale payment plan from all health care facilities.

B

The professional nurse responsible for increasing respect for the individual and awareness of cultural diversity was: A) Harriet Tubman B) Mary Mahoney C) Isabel Hampton D) Mary Adelaide Nutting

B

Three common conditions that affect cognitive function in the elderly are: A) Blindness, hearing loss, and stroke B) Delirium, depression, and dementia C) Cancer, Alzheimer's disease, and stroke D) Stroke, heart attack, and cancer of the brain

B

Which activity would not be expected by the nurse to meet the cultural needs of the client? A) Developing the structure and process for meeting cultural needs on a regular basis and means to avoid overlooking these needs in clients B) Expecting the client's family to keep an interpreter present at all times day and night to assist in meeting the communication needs of the client while hospitalized C) Promoting and supporting attitudes, behaviors, knowledge, and skills to respectfully meet the client's cultural needs despite the nurse's own beliefs and practices D) Ensuring that the interpreter understands not only the client's language but also the feelings and attitudes behind cultural practices to make sure an ethical balance can be achieved

B

Which of the following is the most accurate information to give a nurse during change-of-shift reporting? A) Client refuses to take medications. B) Client reports sharp pain in left anterior knee. C) Client encouraged to consume more fluids. D) Client expressed concern about pending surgery.

B

28. While caring for a child, you identify that additional safety teaching is needed when a young and inexperience mother states that: A. Teenagers need to practice safe sex. B. A 3-year old can safely sit in the front seat of the car C. Children need to wear safety equipment when bike riding D. Children need to learn to swim even if they do not have a pool

B (Car seat required until child is 8 years old or 80 pounds)

33. When evaluating a postthroractomy client with a chest tube, the best method to properly maintain the chest tube would be to: A. Strip the chest tube every hour to maintain drainage B. Place the device below the client's chest C. Double clamp the tube except during assessment D. Remove the tubing from the drainage device to check for proper suctioning

B (never remove tubing)

50. Your client had met the goals set for improvement of ambulatory status. You would now: A. Modify the care plan B. Discontinue the care plan C. Create a new nursing diagnosis that states goals have been met D. Reassess the clients response to care and evaluate the implementation step of the nursing process

B (rationale: Goal has been met)

A client with diabetes is hospitalized with a sore on his foot that has failed to heal. The nurse is gathering a videotape and some printed material on diabetes to begin teaching the client when he calls the nurse asking for something to decrease his pain. In terms of the elements of the communication process, the referent in this situation is: A) The nurse B) The client's pain C) The videotape and printed material on diabetes D) The client's and nurse's sociocultural background

B A referent motivates one person to communicate with another. In this case, sensations and perceptions of pain initiated communication. The videotape and printed material are means of conveying and receiving messages, called channels. The nurse is the receiver, the person who receives and decodes the message. The sociocultural background of the client and nurse are interpersonal variables that influence communication.

The nurse should understand law primarily because the nurse: A) Wants to avoid lawsuits B) Can be an advocate for clients C) Is mandated to review law to keep licensure D) Can protect the hospital from minor lawsuits

B As an advocate for the client, the nurse must make sure that "safe, effective care" is given in conformity with the Nurse Practice Act (NPA). The client is the primary recipient of care and is the most important party in health care relationships. Self, hospital, and physicians are secondary to the outcomes of client care. Nurses should focus on giving correct care to avoid lawsuits. Legal review is a good practice to follow but is not mandated for licensure. The nurse's first responsibility is to the client. Giving proper care will protect an employer from many lawsuits.

According to the Nurse Practice Act (NPA), how are cultural needs addressed? A) There are no references to culture or ethnicity of clients in the NPA. B) Knowledge of cultural aspects is to be incorporated into the plan of care to meet each client's unique needs. C) Cultural needs can be ignored when the client is very ill and about to die, because physical needs are more important at this time. D) Only scientifically based methods of treatment are to be used in nursing; use of other methods or therapies is not the role of the nurse.

B Assessment and implementation of care must consider the total needs of the client, and cultural needs are one of the major areas of assessment. In the Standards of Professional Practice, the nurse is directed to follow all federal, state, and local laws governing the practice of registered nurses, and to provide nursing services without discrimination, regardless of the national origin, race, religion, or health problems of the client served. Cultural needs are incorporated, especially at the time of death, when many religious beliefs and practices of the client and family affect what care is expected from the health care team. A care activity does not have to have a scientific basis for it to be performed. If the client believes that a certain practice will help him or her get better and no harm is expected from the practice, the nurse is expected to support it for the betterment of the client.

The nurse tells the client, "I'm not sure I understand what you mean by 'sicker than usual.' What is different now?" The nurse is using the therapeutic technique of: A) Focusing B) Clarifying C) Paraphrasing D) Providing information

B Clarifying gives the client a chance to be more specific or give more information. Paraphrasing means restating another's message briefly in one's own words. The nurse is not providing information in the remarks given. Focusing is used to bring attention to key concepts or elements in a message.

Which theories describe an orderly process beginning with conception and continuing through death? A) Systems theories B) Developmental theories C) Interdisciplinary theories D) Stress and adaptation theories

B Developmental theories discuss human growth from conception to death. The other options are incorrect.

During the change-of-shift report the night nurse states that a client mentioned having a bad experience with surgery in the past. The nurse was called away and was unable to continue the conversation with the client. The nurse tells the day shift nurse about the comment and notes that the client appears anxious. When the day shift nurse visits the client to clarify the client's bad experience with surgery, the nurse is exhibiting which aspect of critical thinking? A) Integrity B) Discipline C) Confidence D) Perseverance

B Discipline includes completing the task at hand, including assessments (which were not completed on the previous shift). Integrity includes recognizing when one's opinions conflict with those of others and finding a mutually satisfying solution. Confidence is demonstrated in one's presentation and belief in one's knowledge and abilities. Perseverance helps the critical thinker to find effective solutions to client care problems, especially when they have been previously unresolved

Which of the following illustrates the focus of the nurse's interaction during the working phase of the nurse-client helping relationship? A) The nurse says to the client, "Hi, Mr. Owen. My name is Gwen, and I'll be your nurse today." B) The nurse asks the client, "What do you think would help you recover more quickly from your surgery?" C) The nurse asks another nurse while receiving a report, "What did the laboratory report indicate for Mr. Owen?" D) The nurse tells the client, "My shift will be over in about 30 minutes, but I'll see you again tomorrow. You did really well with physical therapy today."

B During the working phase, the nurse helps the client with self-exploration and goal setting. Option 4 illustrates the termination phase, in which the nurse reminds the client that termination is near and evaluates goal achievement with the client. Option 3 exemplifies the preinteraction phase, in which the nurse reviews available data and talks with other caregivers who may have information about the client. Option 1 demonstrates the orientation phase, in which the nurse begins to establish a relationship that initially is superficial using introductions and social talk. The nurse sets the tone for the relationship in a caring manner and clarifies the client's and nurse's roles.

The nursing paradigm identifies four linkages of interest to the nursing profession. These four linkages are: A) Concepts, definitions, relationships, and assumptions B) The person, health, environment/situation, and nursing C) The individual, groups, situations, and interests specific to nursing D) Description, explanation, prediction, and prescription of an interrelationship of nursing

B Identified linkages of a nursing paradigm are the person, health, environment/situation, and nursing itself. Concepts, definitions, relationship, and assumptions are components of a theory. The individuals, groups, situations, and interests specific to nursing are potential subjects for middle-range theories. Description, explanation, prediction, and prescription of an interrelationship of nursing are purposes of nursing theory.

Even though the nurse may obtain the client's signature on a form, obtaining informed consent is the responsibility of: A) The client B) The physician C) The nursing student D) The supervising nurse

B Informed consent is part of the health care provider relationship. The person responsible for performing the given procedure has the responsibility for obtaining the consent.

A nursing student gives herself positive messages regarding her ability to do well on a test. This is an example of what level of communication? A) Public B) Intrapersonal C) Interpersonal D) Transpersonal

B Intrapersonal communication is a powerful form of communication that occurs within an individual. Interpersonal communication takes place between people. Transpersonal communication occurs within a person's spiritual domain. Public communication is interaction with an audience.

A pregnant client mentally rehearses giving birth in her mind. This is an example of: A) Metacommunication B) Intrapersonal communication C) Interpersonal communication D) Transpersonal communication

B Intrapersonal communication is also called self-talk, self-instruction, and inner thought. Self-instruction can provide mental rehearsal for difficult tasks, such as labor and delivery, so individuals can deal with them more effectively. Interpersonal communication is one-to-one interaction that often occurs face to face. Transpersonal communication is interaction that occurs in a person's spiritual domain, such as through prayer or meditation. Metacommunication is looking at the deeper meaning of what is being said.

The Health Insurance Portability and Accountability Act (HIPAA) of 1996 provides clients with basic rights pertaining to their medical records. Which of the following is a violation of HIPAA provisions? A) Discussing client conditions in the nursing report room at the change of shift B) Posting daily nursing care information along with the medical condition of clients on a message board in the client's room C) Allowing nursing students to review client charts before caring for clients to whom they are assigned D) Releasing client information regarding terminal illness to the family when the client has given permission for information to be shared

B Posting information including the medical condition of clients on a message board is a violation of HIPAA provisions. This information should be kept confidential and should not be placed in a location where visitors might view it. Option 4 is not a privacy violation because the client has given permission for the family to have the information. Options 1 and 3 represent "need to know" situations. In option 1, the information is being shared in a private setting.

A common age-related change in auditory acuity is called: A) Presbyopia B) Presbycusis C) Hypertrophy D) Calcification

B Presbycusis is the loss of sensitivity to high-frequency tones that occurs with aging. Presbyopia is the loss of visual accommodation for near vision in the older adult. Calcification is the hardening of previously soft or flexible structures. Hypertrophy is an enlargement.

Most litigation involving hospital care is related to situations in which: A) The nurse abandons the clients when going to lunch. B) The nurse follows an order that is incomplete or incorrect. C) The nurse documents the physician's blame when a mistake is made. D) The supervisor watching a new employee checks his or her skill level.

B The nurse is responsible for clarifying all orders that are illegible, unreasonable, unsafe, or incorrect. When a nurse leaves for lunch or leaves the floor, another person is notified and takes responsibility for the management of the clients left on the floor. Chart keeping is not meant to be punitive and to lay blame on the physician by describing everything in great detail. The supervisor is responsible for knowing the skills of every worker who provides care under that supervisor's authority.

When working with an older adult, the nurse should remember to avoid: A) Touching the client B) Shifting from subject to subject C) Allowing the client to reminisce D) Asking the client how he or she feels

B The nurse should avoid shifting from subject to subject, because it can create confusion. All individuals require touch. Allowing older adults to reminisce can be helpful and therapeutic. Asking a client how he or she feels is a method of opening communication.

Which of the following statements about culture is correct? A) Subcultures are cultures that are inferior to others. B) Culture is not inherited but is a result of socialization. C) Cultural needs are the same as racial needs because they originate from biological traits that are unique to that group. D) Cultural conflict is the inability of a person to decide which cultural pathway to follow when exposed to multiple cultures.

B The process of gaining culture is a process of learning through exposure and incorporation of cultural elements into one's own belief system. Racial traits are genetically passed on and include such things as bone structure, tendencies to develop various disorders, and recognizable characteristics (texture of hair, shape of nose, etc.). Cultural concepts are not genetic but are learned and do not depend on biological traits. Subcultures are smaller recognizable groups within a larger societal group that have their own unique characteristics. Cultural conflict is the conflict between two values or beliefs that a person might have.

The nursing theory that emphasizes the delivery of nursing care for the whole person to meet the physical, emotional, intellectual, social, and spiritual needs of the client and family is: A) Rogers' theory B) Abdellah's theory C) Henderson's theory D) Nightingale's theory

B The question describes the nursing theory developed by Fay Abdellah and others. Rogers' theory considered the individual as an energy field existing within the universe. Henderson's theory defines nursing as "assisting the individual, sick, or well, in the performance of those activities that will contribute to health, recovery, or a peaceful death." Nightingale viewed nursing as providing fresh air, light, warmth, cleanliness, quiet, and adequate nutrition.

Which of the following is subjective information to be entered in the client's medical record? A) Skin warm and dry. B) Pain intensity 8 out of 10. C) Breath sounds clear to auscultation. D) Amber urine in sufficient quantities.

B) Pain intensity 8 out of 10. Pain is purely a subjective phenomenon. Although the pain intensity rating is an objective number, it depends on the client's report. The other options are objective data.

A nurse is working in an acute care hospital that uses a case management model. About which of the following activities should the nurse communicate with the case manager? (Select all that apply.) A) Management of a client transfer to the radiology department B) Coordination of a client transfer to the step-down rehabilitation unit C) Follow-up after a client's discharge to evaluate whether needs have been met D) Permission for a family to bring in special food for a client

B, C The case manager coordinates the efforts of all disciplines to achieve the most efficient and appropriate plan of care for the client, with a focus on discharge planning. Therefore, coordination of transfer to a step-down rehabilitation unit and follow-up after discharge to evaluate that needs have been met are the correct answers.

Which of the following statements is true about evidence-based practice? (Select all that apply.) Evidence-based practice: A) Is based only on the results of research B) Assists nurses in meeting standards of practice C) Helps nurses solve dilemmas in the clinical setting D) Requires nurses to review and critique research and practice findings

B, C, D Evidence-based practice helps nurses to solve dilemmas in the clinical setting because it combines scientific research with clinical expertise and local values. Evidence-based practice does require nurses to review and critique research and practice findings. Nurses are expected to always meet the standards of practice.

A nurse works on a cardiac unit. The nurse is taking care of a client who recently underwent coronary bypass surgery. Which of the following represent legal sources of standards of care nurses use to deliver safe health care? (Select all that apply.) A) Information provided by the head nurse B) Regulations identified in the Joint Commission manual C) Policies and procedures of the employing hospital D) Nurse Practice Act of the state in which the nurse is working E) American Nurses Association standards of nursing practice

B, C, D, E All except the information provided by the head nurse provide legal guidelines for minimum acceptable nursing care.

A nurse is sued for failure to monitor a client appropriately. Which statements are correct about professional negligence lawsuits? (Select all that apply.) A) The nurse is the plaintiff. B) The person filing the lawsuit has the burden of proof. C) The defendant must prove injury, damage, or loss. D) The plaintiff must prove that a breach in the prevailing standard of care caused an injury.

B, D The plaintiff (the person filing the suit) has the burden of proof and must prove that a breach in the prevailing standards of care caused an injury. The nurse would be a defendant in this case. The plaintiff, not the defendant, must prove injury, damage, or loss.

45.When caring for a client who has multiple health problems and related medical diagnoses, nurses can best perform nursing diagnoses and nursing interventions by developing a: A. Critical pathway B. Nursing care plan C. Concept map D. Diagnostic label

C

56. You receive and order to being enteral tube feedings, The first step you must take is to: A. Place the client in a prone position B. Irrigate the tube with normal saline C. Check to see that the tube is properly placed D. Introduce a small amount of fluid into the tube before the feeding

C

58. A health care provider may suspect that a client is experiencing urinary retention when the client has: A. Large amounts of voided cloudy urine B. Pain in the suprapubic region C. Small amounts of urine voided 2-3 times per hour D. Spasms and difficulty during urination

C

An 18-year-old woman is in the emergency department with fever and cough. The physician asks the nurse to measure vital signs, auscultate lung sounds, listen to heart sounds, determine the level of comfort, and collect blood and sputum samples for analysis. The nurse is performing what aspect of practice? A) Diagnosis B) Evaluation C) Assessment D) Implementation

C

Before performing a procedure for the first time at a new agency, the travel nurse: A) Refuses to perform the procedure B) Asks the charge nurse how to perform the procedure C) Reads about the procedure in the policy and procedure manual D) Performs the procedure as at the agency where the nurse previously worked

C

Kyphosis, a change in the musculoskeletal system, is characterized by: A) Decreased bone density in the vertebrae and hips B) Increased risk for pathological stress fractures in the hip and wrist C) Changes in the configuration of the spine that affects the lungs and thorax D) Calcification of the bony tissues of the long bones, such as those in the legs and arm

C

Nurses have the responsibility to dispel myths and replace stereotypes of older adults with accurate information. The nurse knows that most older adults: A) Are confused B) Are forgetful and rigid C) Have a reduced ability to respond to stress D) Are unable to understand and learn new information

C

Socialization of a 6-year-old child from Mexico into the Mexican culture is best described as: A) Assimilation B) Biculturalism C) Enculturation D) Acculturation

C

The client is a 65-year-old overweight woman with multiple medical diagnoses, including diabetes mellitus type 2, hypertension, and residual right-sided weakness resulting from a previous cerebrovascular accident. What tool should be used to plan her care? A) Care plan B) Care map C) Concept map D) Critical thinking

C

The document that developed goals and objectives to meet the health of the public is known as: A) Notes on Nursing B) Last Acts Campaign C) Healthy People 2010 D) Nursing Principles and Practice 2010

C

The dominant values in American society of individual autonomy and self-determination: A) Do not have an effect on health care B) Rarely have an effect on those of other cultures C) May be in direct conflict with the values of diverse groups D) May hinder the ability to gain admission to hospice programs

C

The nurse addressing cultural needs during the postpartum period knows that which of the following statements is correct? A) Hindu mothers prefer bathing and sitz baths to feel clean after delivery and prefer liquid diets for several days after delivery. B) In Western medicine it is common practice to perform religious rituals, such as a cleansing bath, before sexual relations are resumed after delivery. C) Members of non-Western cultures have fewer problems with postpartum depression because attention is given to the mother's recovery for a longer period of time. D) Eastern cultures, such as the Chinese, encourage activity and exercise by the mother soon after the delivery and a return to social involvement as soon as possible.

C

The nurse is assessing the urinary history of a middle-aged married woman. The nurse asks her if she gets up at night. She replies, "Yes." What other question should the nurse ask? A) "How many times do you get up at night?" B) "How long have you been getting up at night?" C) "Why do you get up at night?" D) "How easily do you go back to sleep after you get up?"

C

The nurse is caring for a client with end-stage lung disease. The client wants to go home on oxygen therapy and be comfortable. The family wants the client to undergo a new surgical procedure. The nurse explains the risk and benefits of the surgery to the family and discusses the client's wishes with the family. The nurse is acting as the client's: A) Manager B) Educator C) Advocate D) Caregiver

C

The older adult is at risk for falls for various reasons. To help prevent falls the nurse may: A) Reduce background noise. B) Use vest restraints when the older adult goes to bed. C) Instruct the older adult to use a night light in the bathroom. D) Instruct the older adult to place throw rugs on the floor to remove glare.

C

Which of the following is the biggest consumer of health care? A) Hospitals B) Businesses C) Federal government D) Private insurance companies

C

22. A nurse is working with a terminally ill adult client. The nurse decides to tell the adult children that they need to decide how to advice their father about taking analgesic during the terminal phase of his illness. This step of processing an ethical dilemma is: A. Articulation of the problem B. Evaluation of the action C. Negotiation of the outcome D. Determination of the values surrounding the problem

C (Outcome: quality of life for the duration of end-of-life)

The nurse completes the standard orders on a client's first day postoperatively. The instrument that is used to coordinate the client's care is: A) A Medicare plan B) A discharge plan C) A critical pathway D) Standard nursing care

C A critical pathway is a multidisciplinary treatment plan with interventions prescribed within a structured framework. A discharge plan includes an assessment and anticipation of the client's needs. Medicare is a federal health insurance plan for those 65 years of age and older. Standard nursing care is the minimum care to be given to a client.

The nurse notes that an advance directive is in the client's medical record. Which of the following statements represents the best description of guidelines a nurse would follow in this case? A) A durable power of attorney for health care is invoked only when the client has a terminal condition or is in a persistent vegetative state. B) A living will allows an appointed person to make health care decisions when the client is in an incapacitated state. C) A living will is invoked only when the client has a terminal condition or is in a persistent vegetative state. D) The client cannot make changes in the advance directive once the client is admitted into the hospital.

C A living will directs the client's healthcare in the event of a terminal illness or condition. A durable power of attorney is invoked when the client is no longer able to make decisions on his or her own behalf. The client may change an advance directive at any time.

A theory is a set of concepts, definitions, relationships, and assumptions that: A) Formulates legislation B) Explains a phenomenon C) Measures nursing functions D) Reflects the domain of nursing practice

C A theory is a set of concepts, definitions, relationships, and assumptions that explains a phenomenon. Theories do not formulate legislation, measure nursing functions, or reflect any domain of nursing practice.

The nurse demonstrates active listening by: A) Agreeing with the client B) Repeating everything the client says to clarify C) Assuming a relaxed posture and leaning toward the client D) Smiling and nodding continuously throughout the interview

C Active listening means being attentive to what the client is saying both verbally and nonverbally. Assuming a relaxed posture and leaning toward the client facilitates listening. Agreeing with the client does not facilitate communication. Repeating everything the client says can become distracting. Smiling and nodding continuously is not responding to what the client says at all times.

The nursing process is an example of an open system. An open system: A) Is universal and dynamic B) Represents a relationship between two concepts C) Interacts with the environment by exchanging information D) Is a process through which information is returned to the system

C An open system is defined as a system that interacts with the environment, exchanging information between the system and the environment.

The multidisciplinary care model used to move clients efficiently from admission to discharge is known as: A) Team nursing B) Nursing process C) Case management D) Interdisciplinary care

C Case management is a model of organizing care in which the case manager monitors, directs, and advises the nursing care personnel on specific care issues and the progress of a client. In team nursing, care might be provided by groups composed of registered nurses, licensed practical nurses, and possibly assistive personnel. Nursing process is used to plan the nursing care for a client. Interdisciplinary care is care provided by a team whose members come from a variety of disciplines.

If the nurse is working with a client who has expressive aphasia, it would be most helpful for the nurse to: A) Ask open-ended questions. B) Speak loudly and use simple sentences. C) Allow extra time for the client to respond. D) Encourage a family member to answer for the client.

C For clients with aphasia, the nurse should be sure to allow extra time for the client to respond. Asking open-ended questions is important, but these questions need to be developed based on the client's personal communication ability. Speaking loudly is not necessary for a client with a diagnosis of aphasia. The client should be encouraged to answer questions himself or herself and not expect others to answer for the client, even if it takes longer for the client to do so.

The nurse is complying with the provisions of which act when the nurse requests that the client give permission to copy the client's medical reports for the insurance company? A) Uniform Anatomical Gift Act B) Mental Health Parity Act C) Health Insurance Portability and Accountability Act (HIPAA) D) Americans With Disabilities Act (ADA)

C HIPAA was enacted to control information distribution and allow the client to take control of where information is sent. Permission is now required for insurance companies to receive parts of charts that are relevant to their needs. The Uniform Anatomical Gift Act regulates permission to obtain organs for donation and prevents illegal or immoral removal of organs for sale or profit. The Mental Health Parity Act stipulates how permission should be obtained and the wishes of the client considered in providing medical management of mental illness when mental clarity is not longer present. Americans With Disabilities Act is not the correct choice.

The nurse is giving discharge instructions to a client with newly diagnosed diabetes. The nurse discusses with the client what the dietary intake should be. This is an example of which health care service? A) Tertiary care B) Restorative care C) Health promotion D) Illness prevention

C Health promotion includes dietary counseling. Blood glucose monitoring at the pharmacy is an example of illness prevention. Restorative care is care of a client who, for instance, is recovering from complications of diabetes. Any diagnostic procedure or tests completed in the hospital would be examples of such care.

To practice in today's health care environment, nurses need a strong scientific knowledge base in nursing and other disciplines, such as the physical, social, and behavioral sciences. This relates to which of the following? A) Systems theories B) Developmental theories C) Interdisciplinary theories D) Health and wellness model

C Interdisciplinary theories provide a systematic view of a phenomenon. Developmental theories, health and wellness theories, and systems theories are examples of other types of theories.

A client who is confused is left alone in bed with the side rails down and the bed in a high position, and the client falls and breaks a hip. In legal terms, what has occurred? A) Assault B) Battery C) Negligence D) Civil tort

C Knowing what to do to prevent injury is a part of the standard of care that nurses must follow. Negligence is conduct that falls below the standard of care that protects others against unreasonable risk of harm. Assault is the threat to engage in harmful or offensive contact. Battery is the actual unlawful touching of another, whether threat of harm is included or not. Intentional touching without permission or consent is not lawful. A civil tort is a civil wrong committed against a person or property. An example of a tort involving property would be to lose the client's dentures by misplacing them.

Which task is it not appropriate for a professional nurse to delegate to assistive personnel? A) Ambulate a client B) Complete a bed bath C) Obtain a sterile urine specimen D) Complete the intake and output (I&O) record

C Obtaining a sterile specimen requires insertion of a catheter, a procedure that must be performed by a licensed nurse. Therefore, this would not be an appropriate task to delegate to an assistive person. Assistive personnel would be able to ambulate a client, give a bed bath, and add to the I&O record.

Nursing theories focus on the phenomena of nursing and nursing care. Which of the following is true of phenomena? A) They are aspects of reality that can be consciously sensed or experienced. B) They convey the general meaning of concepts in a manner that fits the theory. C) They are statements that describe concepts or connect two concepts that are factual. D) They are mental formulations of an object or event that come from individual perceptual experience.

C Phenomena are defined as aspects of reality that can be consciously sensed or experienced.

Which of the following statements about prescriptive theories is accurate? A) They describe phenomena. B) They have the ability to explain nursing phenomena. C) They reflect practice and address specific phenomena. D) They provide a structural framework for broad abstract ideas.

C Prescriptive theories address nursing interventions for a phenomenon and predict the consequence of a specific nursing intervention. Descriptive theories describe the phenomena, speculate on the reason the phenomena occur, and predict nursing phenomena. Grand theories are broad and complex and provide a structural framework for broad, abstract ideas about nursing.

The type of theory that tests the validity and predictability of nursing interventions is: A) A grand theory B) A descriptive theory C) A prescriptive theory D) A middle-range theory

C Prescriptive theory addresses nursing interventions and predicts the consequence of a specific nursing intervention. Middle-range theories are limited in scope, less abstract than grand theories, address specific phenomena or concepts, and reflect practice. Descriptive theories describe phenomena, speculate as to why the phenomena occur, and describe the consequences of phenomena. Grand theories are broad and complex.

A nursing instructor notices that a student nurse is showing a lack of professionalism when the student: A) Accepts responsibility for an error he made in documentation B) Arrives on time and is clean and neat, wearing no perfume or cologne C) Shares personal information about his assigned client with other students not involved in the client's care D) Knocks on the door before entering and says, "Hello, Mr. Smith. I am Bill Johnson, and I'll be your student nurse today."

C Sharing personal information about others violates nursing ethical codes and practice standards. Team members directly involved in the client's care should be given only relevant information about the client's status. To practice courtesy, the nurse knocks on doors before entering and uses self-introduction. A professional is expected to be clean, neat, well groomed, conservatively dressed, and scent and odor free. Being on time, organized, well prepared, and equipped for the responsibilities of the nursing role also communicate one's professionalism. Professional nurses make choices and accept responsibility for the outcome of their actions.

Each science has a domain, which is the perspective of the discipline. This domain: A) Represents the recipients of the benefits of the science or discipline B) Is a model that explains the linkage of science, philosophy, and theory that is accepted and applied by the discipline C) Describes the subject, central concepts, values and beliefs, phenomena of interest, and central problems of the discipline D) Is a dynamic state of being in which the developmental and behavioral potential of the individual is realized to the fullest

C The domain contains the subject, central concepts, values and beliefs, phenomena of interest, and the central problems of the discipline. A paradigm is a model that explains the linkage of science, philosophy and theory that is accepted and applied by the discipline.

Maslow's hierarchy of needs is useful to nurses, who must continually prioritize a client's nursing care needs. The most basic or first-level needs include: A) Self-actualization B) Love and belonging C) Air, water, and food D) Esteem and self-esteem

C The first level of Maslow's hierarchy of needs includes the need for air, food, and water—basic elements of survival. Love and belonging are on the second level, esteem and self-esteem are on the fourth level, and self-actualization is the final level.

Which statement about older adults in the United States is correct? A) Most older adults live in institutional settings. B) The proportional number of adults older than age 85 is decreasing. C) The number of older adults is rising because of the increase in the average life span and the aging of the baby boom generation. D) Some 75% of adults aged 65 to 74 years report disability or limitation in activities due to chronic disease.

C The number of older adults is expected to reach 70 million by 2030 because of a lengthening of the average life span, the aging of the baby boom generation, and growth in the population segment older than age 85. Most older adults live in noninstitutional settings. In 2000, only 4.5% of all older adults resided in institutional settings. Both the number of individuals older than age 85 and the percentage of older adults in minority groups are increasing, not decreasing, in the United States. Most older people remain functionally independent despite the increasing prevalence of chronic disease. Some limitation on activities caused by chronic conditions was reported by 28.8% of adults aged 65 to 74 and 50.6% of adults older than age 75.

The nurse asks a client how she feels about impending surgery for breast cancer. Before initiating the discussion the nurse reviewed information about loss and grief in addition to therapeutic communication principles. The critical thinking component involved in the nurse's review of the literature is: A) Experience B) Problem solving C) Knowledge application D) Clinical decision making

C The nurse sought appropriate information to be able to communicate more knowledgeably with the client. Experience is acquired through clinical learning situations. Problem solving is a series of steps to resolve a problem. Clinical decision making is a process in which critical thinking steps are followed for problem resolution.

Benner defines this nurse as able to anticipate nursing care and to formulate long-range goals; this nurse is given the title: A) Expert nurse B) Proficient nurse C) Competent nurse D) Advanced beginner

C The nurse who has held the same position for 2 to 3 years and understands the specific area and client population is termed a competent nurse. The expert is a nurse with diverse experience who can focus on a specific problem and offer multidimensional solutions. The proficient nurse has more than 2 to 3 years' experience and applies knowledge and experience to a situation. The advanced beginner nurse has at least some level of experience.

When assessing the older adult, the nurse should know which findings represent common physiological changes associated with aging and which are abnormal findings. A normal and common physiological change is: A) Urinary incontinence B) Increased saliva production and small intestine motility C) Increase in the time it takes for the heart rate to return to baseline after exercise D) Cold feet caused by a decrease in muscle mass and a decrease in the number of neurons

C The older adult's body tries to compensate for decreased cardiac output by increasing the heart rate during exercise. After exercise, however, it takes longer for the older adult's heart rate to return to baseline. Older adults have decreased saliva flow and slowing of peristalsis. Decreased motility of the small intestine may increase the older adult's risk for developing constipation. Urinary incontinence is an abnormal condition, not a normal physiological response to aging. The older adult does have decreased muscle mass and strength; however, cold feet would more likely be the result of decreased cardiac output and decreased circulation in the lower extremities as evidenced by weaker peripheral pulses in the feet.

A nurse should consider zones of personal space and touch when caring for clients. If the nurse is taking the client's nursing history, she should: A) Sit next to the client B) Be 4 to 12 feet from the client C) Be 18 inches to 4 feet from the client D) Be 12 inches to 3 feet from the client

C The personal zone is 18 inches to 4 feet and is best when the nurse is taking a client's history. The intimate zone is 0 to 18 inches, and the nurse is in this zone when performing assessment. The social zone is 9 to 12 feet and is used when making rounds with a physician. The public zone is 12 feet or more.

Which of the following was most significant in influencing competition in health care costs? A) Medicare and Medicaid B) Diagnosis-related groups C) Prospective payment system D) Managed care organizations

C The prospective payment system is one of the most significant factors influencing payment for health care. The prospective payment system groups payments into diagnosis-related groups for Medicare and Medicaid clients. Managed care organizations are systems in which there is administrative control over primary health care services for a defined client population.

The legal definition of death that facilitates organ donation is cessation of: A) Pulse B) Respirations C) Functions of the entire brain D) Circulatory and respiratory functions

C The whole-brain standard of death requires irreversible cessation of all functions of the entire brain, including the brainstem. This definition allows for the recovery of organs for transplantation. Individuals who are not donors typically are pronounced dead when there is a total cessation of circulatory and respiratory functions. The absence of pulse and respirations must occur together to meet the legal definition of death.

There is a contemporary move toward addressing nursing as a science or as evidenced-based practice. This suggests that: A) One theory will guide nursing practice. B) Scientists will make nursing decisions. C) Theories will be tested to describe or predict client outcomes. D) Nursing will base client care on the practice of other sciences.

C Theories will be tested to describe or predict client outcomes as nursing is addressed as a science and an art. Scientists will not make nursing decisions, and nursing will base client care on the practice of nursing science, which will be guided by multiple theories.

Which of the following is true about aging? A) Genetic changes are solely responsible. B) The client's environment is the main factor. C) There is no single theory that explains aging. D) The presence of disease is what causes a decline in function.

C There is no single accepted theory about aging. There are stochastic and nonstochastic theories of aging, some of the ideas of which are reflected in the other answer options, but no single theory fully explains all aspects of aging.

The client's health insurance changed, and instead of having a limited number of physicians from whom to choose, the client is voluntarily enrolled in a plan in which medical care is provided by a special group of caregivers. This arrangement is known as: A) Medicare B) Private insurance C) Managed care organization (MCO) D) Preferred provider organization (PPO)

C This is the description of an MCO. In a PPO, choice of care providers is limited to those listed in the group. Medicare is a federally funded national health insurance program. Private insurance is a traditional fee-for-service plan.

The nurse may facilitate verbal communication with clients by: A) Using words that can have several meanings B) Using medical terminology to ensure accuracy C) Using short sentences that express an idea simply and directly D) Speaking slowly and deliberately and allowing long pauses in the conversation

C Verbal communication should be clear and brief. Fewer words result in less confusion. Communication that is simple, brief, and direct is more effective. Medical jargon may sound like a foreign language to clients unfamiliar with the health care setting and should be used only with other health team members. Nurses should carefully select words that cannot be easily misinterpreted, especially when explaining a client's medical condition or therapy. Speaking slowly and deliberately can convey an unintended message. Long pauses and rapid shifts to another subject may give the impression that the nurse is hiding the truth.

Which of the following is an open-ended question the nurse might use when interviewing a client? A) "Do you have any concerns right now?" B) "Is your family worried about your being in the hospital?" C) "What do you mean when you say, 'I don't feel quite right'?" D) "How many times do you get up to go to the bathroom at night?"

C) "What do you mean when you say, 'I don't feel quite right'?" The way the nurse asks question 3 allows the client to respond completely and with more than a one-word answer. The other options allow the client to respond with one word and make it unlikely that the client will give additional information.

What type of interview technique is the nurse using when the nurse asks the question, "Do you have pain or cramping?" A) Active listening B) Open-ended questioning C) Closed-ended questioning D) Problem-oriented questioning

C) Closed-ended questioning The example is a closed-ended question which the client can answer with a one-word reply. Open-ended questions allow the client to answer with more information. The other options are not correct.

BLEEDING PRECAUTION PATIENT. THESE PATIENT WHO ARE ON: .

Coumadin/ Heparin , DIC, Hemophila, Chemo, Problem w/ bone marrow, Cancer, Liver disease, On aspirin or NSAIDS

11. A client you are assisting has fallen in the shower. You must complete an incident report. The purpose if an incident report is to: A. Exchange info among health care members B. Provide info about clients on one unit to another unit C. Ensure proper care for the client D. Aid in the hospitals quality improvement program

D

12. You have delegated vital signs to assistive personnel. The assistant informs you that the client has finished a bowl of hot soup. The nurse's most appropriate advice would be to: A. Take a rectal temp B. Take the oral Temp as planned C. Advise the client to drink a class of cold water D. Wait 30 minutes and take an oral temperature

D

20. Your client is about to undergo a controversial orthopedic procedure. The procedure may cause periods of pain. Although nurses agree to do no harm, this procedure may be the client only treatment choice. The example describes the ethical principle of: A. Autonomy B. Fidelity C. Justice D. Nonmaleficence

D

36. During round on the night shift, you note that a client stops breathing for 1 to 2 min several times during the shift. This condition is known as: A. Cataplexy B. Insomnia C. Narcolepsy D. Sleep apnea.

D

39. The nursing process organized your approach while delivering nursing care. To provide the best professional care to the clients, nurses need to incorporate nursing process and: A. Decision making B. Problem solving C. Intellectual standards D. Critical thinking skills

D

4. Technological advances in health care: A. Make the nurses job's easier B. Depersonalize bedside patient care C. Threaten the integrity of the health care industry D. Do not replace sound personal judgement

D

48. You are writing a care plan for a newly admitted client, which one of the outcome statement is written correctly? A. The client will eat 80% of meals B. The nursing assistant will set up the client for a bath every day C. The client will have improved airway clearance by June 5 D. The client will identify the need to decrease dietary intake of fiber by June 5

D

49. You have finished with several nursing interventions o evaluate interventions you will need to examine the : A. Appropriateness of the intervention and the correct application of the implementation process B. Nursing diagnoses to ensure they are not medical diagnoses C. Care planning process for errors in the other health care team members judgments D Interventions of each nurse to enable the nurse manager to correctly evaluate performance

D

5. You are invited to attend the weekly unit client care conference. The staff discussed client care issues. This type of communication is: A. Public B. Intrapersonal C. Transpersonal D. Small group

D

55. A 22 year old new mother is breast feeding. You ask her if she is taking the correct amounts of nutrients. You know the young mother understand the dietary guideline when she states : A. "I am not concerned with what I am eating." B. "I am taking the vitamin doses according to the TV commercials." C. "I am taking one multiple vitamin a day and eating whatever else I feel like." D. "I am making eating choices according to the recommended dietary allowances and intakes."

D

63. A post op client arrives at an ambulatory care center and state "I am not feeling good." Upon assessment, you note an elevated temperature. An indication that the wound is infected would be: A. It has no odor B. A culture is negative C. The edges reveal the presence of fluid D. It shows purulent drainage coming from the incision site

D

64. A surgical wound required a hydrogel dressing. The primary advantage of this type of dressing is that it provides: A. An absorbent surface to collect wound drainage B. Decreased incidence of skin maceration C. Protection from the external environment D. Moisture needed for wound healing

D

9. A young girl is having problems urinating postoperative. You remember that children may have trouble voiding: A. In bathrooms other than their own B. In a urinal C. While lying in bed D. In the presence of person other than their parents

D

A client is wheezing and short of breath. The physician orders a medicated nebulizer treatment now and in 4 hours. The nurse is providing what aspect of care? A) Planning B) Evaluation C) Assessment D) Implementation

D

For a nursing student to enhance cultural awareness, the student will need to make an in-depth self-examination of: A) Motivation and commitment to caring B) Social, cultural, and biophysical factors C) Engagement in cross-cultural interactions. D) Background, including recognition of biases and prejudices

D

On entering a client's room during change-of-shift rounds, the nurse notices that the client and spouse have their backs turned to each other, and both have their arms folded across their chests. The best action for the nurse to take at this time is to: A) Introduce himself or herself and begin discharge teaching. B) Proceed with the tasks the nurse was intending to perform. C) Say nothing and leave quickly, closing the door behind. D) Ask the client and spouse if they need some time alone right now.

D

Sexuality is recognized as a factor in the care of older adults, thus: A) The need to touch and be touched is decreased. B) A decrease in an older adult's libido does occur. C) Any expression of sexuality should be discouraged. D) All older adults, whether healthy or frail, need to express sexual feelings.

D

Transcultural nursing involves: A) Working in another culture to practice nursing within the limitations of that culture B) Ignoring all cultural differences to provide the best generalized care to all clients C) Combining all cultural beliefs into a practice that takes a nonthreatening approach to minimize cultural barriers and achieve equality of care for all clients D) Using a comparative study of cultures to understand similarities and differences across human groups to provide specific individualized care that is culturally appr

D

Which of the following statements related to cultural conflicts is an accurate definition? A) Cultural ignorance is a refusal to accept another person's beliefs. B) Ethnocentrism is a belief that one's own way of life is inferior to that of others. C) Cultural blindness is the ability to see that there are differences among people. D) Discrimination is bias against people because of age, gender, color, race, or beliefs.

D

17. A postoperative client is received morphine sulfate via PCA. The nurse assess that the client's respirations are depressed. The effects of the morphine sulfate can be classified as: A. Allergic B. Idiosyncratic C. Therapeutic D. Toxic

D (Depressed = less than 12, PCA = Patient-controlled analgesia)

An APN is pursuing a job change. Which of the following positions would the APN be unable to fill without meeting additional criteria? A) Case manager B) Nurse manager C) Nurse educator D) Certified registered nurse anesthetist

D Additional training in anesthesia medicine would be required to be a certified registered nurse anesthetist.

As an art, nursing relies on knowledge gained from practice and reflection on past experiences. As a science, nursing relies on: A) Experimental research B) Nonexperimental research C) Physician-generated research D) Scientifically tested knowledge

D As a science, nursing draws on scientifically tested knowledge applied in the practice set

Which of the following is an example of a positive outcome of a nurse-health team relationship? A) The nurse becomes an effective change agent in the community. B) The nurse better understands the family dynamics that affect the client. C) The nurse better appreciates what the client perceives as meaningful from the client's perspective. D) The nurse receives encouragement and support from co-workers to cope with the many stresses associated with the nursing role.

D Benefits of positive nurse-health team relationships include building morale and strengthening bonds so that team members can help one another cope with the stresses of working in the health care field. Option 1 is a positive outcome of an effective nurse-community relationship, whereas option 2 describes an outcome of a nurse-family relationship. A benefit of narrative interactions in a nurse-client helping relationship is increased understanding, as in option 3.

The professional nurse can best be said to be engaging in collaboration with others to develop the client's plan of care when the nurse: A) Consults the physician for direction in establishing goals for clients B) Depends on the latest literature to complete an excellent plan of care for clients C) Works independently to plan and deliver care and does not depend on other staff for assistance D) Works with colleagues and clients' families to take advantage of combined expertise in planning care

D Collaboration is teamwork in which individuals in multiple disciplines work together, each contributing his or her expertise to the client's care. The physician will provide medical direction, but teamwork and collaboration require more than just medical direction. Consulting the latest literature can help in planning care, but this is not collaboration. The collaborative team works together to provide care for the client.

A theory is a set of concepts, definitions, relationships, and assumptions or propositions to explain a phenomenon. The purposes of the components of a theory are to: A) Describe concepts or connect two concepts that are factual B) Formulate a perceptual experience to describe or label a phenomenon C) Express the global view about the individual, situations, or factors of interest to a specific discipline D) Describe, explain, predict, and/or prescribe interrelationships among the concepts that define the phenomenon

D Describing, explaining, predicting, and/or prescribing interrelationships among concepts are stated purposes of research.

When a nurse stops to help in an emergency at the scene of an accident, if the injured party files suit and the insurance of the nurse's employing institution does not cover the nurse, the nurse would probably be covered by: A) The nurse's automobile insurance B) The nurse's homeowner's insurance C) The Patient Care Partnership, which may grant immunity from suit if the injured party consents D) The Good Samaritan laws, which grant immunity from suit if there is no gross negligence

D Good Samaritan laws grant immunity from lawsuits if the nurse follows the standards of care that a nurse of his or her experience would prudently have followed. The other answers are incorrect.

Which of the following regulates hospitals to ensure safety in the provision of services, establishes criteria that must be met for a hospital to receive funding from the government, and provides for penalties if guidelines are not followed? A) Board of Nursing Examiners (BNE) B) Nurse Practice Act (NPA) C) American Nurses Association (ANA) D) Americans With Disabilities Act (ADA)

D If a hospital fails to follow ADA guidelines for meeting the special needs of persons with disabilities, the facility loses funding and become ineligible to receive low-income loans or reimbursement of expenses. ADA protects the civil rights of disabled people. Its provisions apply to both hospital clients and hospital staff.

A nurse feels frustrated because she is behind in administering her clients' medications. She comes to the client's bedside hurriedly with a frown on her face and sighs while she is waiting for the client to swallow the medication. The nurse then says brightly, "Isn't it a relaxing day?" The nurse should remember that: A) The client may respond defensively if his or her personal space is threatened. B) Maintaining eye contact during conversation shows respect and willingness to listen. C) Most meaning in conversation is transmitted by spoken words rather than by nonverbal communication. D) When there is incongruity between verbal and nonverbal communication, the receiver usually "hears" the nonverbal message as the true message.

D It has been estimated that approximately 7% of meaning is transmitted by words, 38% is transmitted by vocal cues, and 55% is transmitted by body cues. Nonverbal communication is unconsciously motivated and may more accurately indicate a person's intended meaning than the spoken words. This is what the receiver "hears." Although options 1 and 2 are true, they are not the best statements for explaining incongruity between verbal and nonverbal communication as depicted in the question.

A nurse is meeting a 3 year old for the first time. Communication with the child will be facilitated if the nurse: A) Ignores the child B) Lifts the child onto the nurse's lap C) Distracts the child by clapping the hands D) Kneels down while holding and talking to a teddy bear

D Kneeling down puts the nurse at the child's eye level. Holding and talking to a teddy bear is nonthreatening and allows the child to make the first move in interpersonal contacts. Children are especially responsive to nonverbal messages, and sudden movements, loud noises, or threatening gestures can be frightening. Lifting a child onto the nurse's lap may be very threatening to the child because the nurse is invading the child's personal space. Ignoring the child will not facilitate communication.

Mishel's theory of uncertainty in illness focuses on the experience of clients with cancer who live with continual uncertainty. The theory provides a basis for nurses to assist clients in appraising and adapting to the uncertainty and illness response and can be described as: A) A grand theory B) A descriptive theory C) A prescriptive theory D) A middle-range theory

D Middle-range theories are limited in scope, less abstract than grand theories, address specific phenomena or concepts, and reflect practice. Grand theories are described as broad and complex. Prescriptive theories address nursing interventions and predict the consequence of a specific nursing intervention. Descriptive theories describe phenomena, speculate as to why the phenomena occur, and describe the consequences of phenomena.

The best way for nurses to avoid being liable for negligence might be to do all of the following except: A) Follow standards of care. B) Give safe competent care in a caring manner. C) Document assessments, interventions, and evaluations as soon as possible after performing them. D) Choose not to report someone whose professional behavior the nurse questions.

D Of the actions listed, failure to report a person whose professional behavior is questioned is the only one that would expose the nurse to liability for negligence. The nurse by law and the guidelines of the Nurse Practice Act is required to report any unsafe or unethical professional behaviors that are observed. Ignoring bad behavior does not relieve the nurse of the nurse's duty to report this behavior.

A client says to the nurse, "It was a stupid thing that I did. If I had just stayed home, this car accident wouldn't have happened." The nurse's best response is: A) "So, why did you go out?" B) "Why would you say that?" C) "If I were you, I'd quit worrying about it. You can't change the past." D) "You feel responsible for the accident, as though it could've been prevented."

D Option 4 demonstrates the therapeutic communication technique of paraphrasing. Paraphrasing is restating another's message more briefly using one's own words. Through paraphrasing, the nurse sends feedback that lets the client know that the nurse is actively involved in the search for understanding. Asking for explanations is a nontherapeutic communication technique. Giving one's personal opinion is also nontherapeutic. Asking personal questions that are not relevant to the situation to satisfy the nurse's curiosity is both nontherapeutic and nonprofessional.

When assessing the older adult, the nurse should review the client's achievement of developmental tasks. For the older adult, these may include all of the following except: A) Coping with the loss of the work role B) Accepting himself or herself as aging C) Redefining relationships with children D) Engaging in more introspective, self-focused activities

D Option 4 is not a developmental task of older adults but rather a characteristic of this age group according to the disengagement theory of aging. This psychosocial theory holds that aging individuals withdraw from customary roles and engage in more self-focused activities. Older adults retired from employment outside the home are challenged to cope with the loss of that work role, and their spouses may also need to adjust to role changes. Retired adults may have to find new ways to occupy their time. Older adults face the need to adjust to the physical changes that accompany aging. Acceptance of personal aging does not mean retirement into inactivity, but it does require a realistic review of strengths and limitations. Adult children and aging parents must negotiate the parameters of changed roles.

A client discharged after suffering a stroke is transferred from a tertiary care facility to another facility for additional care to help the client recover and continue to regain function. This type of care facility is known as: A) Home care B) Assisted care C) Extended care D) Restorative care

D Restorative care assists an individual in regaining the maximum possible level of functioning. Home care includes professional and paraprofessional services that are rendered in the home setting. Extended care is intermediate medical or nursing care for individuals with an acute or chronic illness or disability. Assisted care is a setting in which the client is able to function at a higher level of autonomy within a homelike environment but in which care can be given when needed.

A nurse volunteers to take blood pressure measurements after church services. This is an example of which level of health care service? A) Secondary care B) Restorative care C) Health promotion D) Illness prevention

D Taking blood pressure measurements is illness prevention. Health promotion includes activities like exercise classes. Secondary care is often known as traditional care. It would include rehabilitation after a stroke in an individual with a history of elevated blood pressure.

When the nurse signs a form as a witness, the nurse's signature shows that the client: A) Is fully informed and is aware of all consequences of signing B) Was awake and fully alert and not medicated with narcotics at the time of signing C) Was free to sign without pressure D) Has signed that form and the witness saw it being done

D The nurse's signature as a witness indicates only that the person signing the form was indeed the person whose name was on the form. The witness does not have to know if the client was fully informed or not. The witnessing agent is indicating no judgment about the level of cognitive function of the client by signing as witness. The nurse should assess for coercion, but the signature of the witness is not an acknowledgement of having performed such an assessment. If the witness feels that someone is forcing a client to sign, however, then the witness may refuse to sign and the contract is then void (unless a different witness signs).

In a nurse-client helping relationship, the nurse should: A) Problem solve for the client. B) Distort the truth when the client is to receive bad news to protect the client. C) Convey acceptance by always agreeing with the client or approving of the client's decisions or actions. D) Convey nonjudgmental acceptance with a willingness to hear a message or to acknowledge feelings.

D The nurse-client helping relationship is characterized by the nurse's nonjudgmental acceptance of the client. Acceptance conveys a willingness to hear a message or to acknowledge feelings even if the nurse does not agree with the client. Acceptance does not mean the nurse must always agree with the client. The nurse should allow others the opportunity to make choices and help them problem solve, but not make decisions for them. The nurse acts as an advocate, keeping the client informed and providing support in decision making. Lying or distorting the truth violates both legal and ethical standards of practice.

Evidence-based nursing practice is the end result of: A) Prescriptive theory B) Use of practical knowledge C) Application of theoretical knowledge D) Theory-generating and theory-testing research

D The result of theory-generating or theory-testing research is to increase the knowledge base of nursing. As these research activities continue, clients become the recipients of evidence-based nursing care.

The purpose of a utilization review committee is to: A) Review quality, quantity, and cost of care B) Review the utilization of the payment mechanism C) Review reimbursement fees and appropriation of funds D) Review admissions, diagnostic tests, and treatments ordered by physicians

D The utilization review committee reviews admissions, diagnostic procedures, and treatments ordered by physicians. Review of the quality, quantity, and cost of care is more similar to the functions of a professional standards review organization. Review of reimbursement fees and appropriation of funds involves review of diagnosis-related groups. Reviewing the utilization of the payment mechanism is similar to capitation.

Which two factors contribute to the projected increase in the number of older adults? A) Financial success and improved environment B) Greater acceptance of older adults and medical problems C) Improved medication plan and increase in Medicare funding D) Aging of the baby boom generation and the growth of the populati

D Two factors that contribute to the projected increase in the number of older adults are the aging of the baby boom generation and the growth in the number of older adults older than age 85. Financial success is individually felt, although there is a sense of improved environment in the United States. The new Medicare medication plan (Part D) probably has little impact on the projected number of older adults, and Medicare funding is not increasing. There is a greater acceptance of older adults and medical problems, but this does not contribute to the projected increase in the number of older adults.

To help reduce confusion of the older adult at night, the nurse may: A) Provide bright light. B) Give the client a sleep aid in keeping with the physician's orders. C) Place hearing aids in a drawer so the client does not hear all of the environmental noise. D) Make telephone calls to friends or family members to let the older adult hear reassuring voices.

D When confusion varies by time of day or is related to environmental factors, the nurse can use creative, nonpharmacological measures such as making the environment more meaningful, providing adequate light, encouraging use of assistive devices (glasses, hearing aids), or making telephone calls to friends or family members to let older adults hear their voices. Older adults have difficulty adjusting to abrupt changes from dark to light (and the reverse). Bright light may also cause glare, which would make seeing more difficult for the older adult. Interventions to increase ambient light should not increase glare. Hearing aids should be given to older adults so they will be less likely to misunderstand what they hear. Reducing background noise may also help their hearing. Nonpharmacological methods should always be used first. Some sleep aids increase confusion.

The purpose of assessment is to: A) Make a diagnostic conclusion. B) Delegate nursing responsibility. C) Teach the client about his or her health. D) Establish a database concerning the client.

D) Establish a database concerning the client. The purpose of assessment is to establish a database about the client's perceived needs, health problems, and responses to these problems. The data also reveal related experiences, health practices, goals, values, and expectations. The other options are not purposes of assessment.

A nurse assessing a client who comes to the pulmonary clinic asks, "Tell me what medications you are taking for your breathing problem. I see from your last visit that Dr. Russell recommended routine exercise. Can you also tell me how successful you have been in following his plan?" The nurse's assessment covers which of Gordon's functional health patterns? A) Value-belief pattern B) Cognitive-perceptual pattern C) Coping-stress tolerance pattern D) Health perception-health management pattern

D) Health perception-health management pattern The health perception-health management pattern involves the client's self-report of health and well-being, how the client manages his or her health, and knowledge of preventative health practices. The cognitive-perceptual pattern involves sensory-perceptual patterns, language adequacy, memory, and decision-making abilities. The coping-stress tolerance pattern involves the client's ability to manage stress, sources of support, and the effectiveness of the patterns in terms of stress tolerance. The value-belief pattern involves the values, beliefs, and goals that guide the client's choices or decisions.

Assessment data must be descriptive, concise, and complete. In performing an assessment the nurse should not: A) Include subjective data from the client. B) Perform a thorough physical examination. C) Use interpersonal and cognitive skills. D) Include inferences or interpretative statements not supported with data.

D) Include inferences or interpretative statements not supported with data. The nurse should not generalize or form judgments not supported by the collected data. Inferences and interpretive statements must be supported by data. Assessments do include conducting a thorough physical examination, using interpersonal and cognitive skills, and obtaining subjective data from the client.

A client who is alert and awake is being transferred to another hospital with a copy of his medical records. Before the transfer the nurse must: A) Ask the hospital lawyer if this requires approval from the risk management department. B) Discuss the need to copy the medical records with the client's family. C) Be certain that the physician writes an order for the record to be copied. D) Obtain written permission to copy the medical records for the receiving hospital.

D) Obtain written permission to copy the medical records for the receiving hospital. Obtaining permission to copy the records demonstrates the nurse's understanding of the provisions of the Health Insurance Portability and Accountability Act (HIPAA). Discussing medical records with the client's family is inappropriate because the client's family does not make the decision for a client who is capable of making his own decision. Policies and procedures would already be in place for the nurse with regard to copying medication records. It is not necessary to call the hospital lawyer. Copying a client's medical record does not require

During data clustering, a nurse: A) Provides documentation of nursing care B) Reviews data with other health care providers C) Makes inferences about patterns of information D) Organizes cues into patterns that lead to identification of nursing diagnoses

D) Organizes cues into patterns that lead to identification of nursing diagnoses During data clustering, the nurse organizes cues into patterns that indicate individualized nursing diagnoses and identify collaborative problems. The other options are incorrect.

2. The NLN and the ANA are professional organizations that deal with A. Nursing issues of concern B. Political and professional issues affecting health care C. Financial issues affecting health care D. All of the above

D. All of the above

34. A client is diaphoretic and has an oral temperature of 104 F, These are classic signs of: A. ADH deficit B. Extracellular fluid loss C. Insensible water loss D. Sensible water loss

D. Sensible water loss (Sensible water loss usually has to do with sweating. It means that you're aware of it. Insensible water loss is something like expiratory loss)

Answer: B Sitting upright provides full expansion of the lungs and provides better visualization of the symmetry of upper body parts. The lateral recumbent position aids in detecting heart murmurs. The dorsal recumbent position is used for abdominal assessment because it allows relaxation of abdominal muscles. The supine position provides easy access to pulse sites.

The nurse should assist the client to a sitting position to provide the best position to examine which of the following? A) Heart B) Lungs C) Abdomen D) Pulse sites

Answer: C To measure pulse deficit the nurse and a colleague assess the radial and apical pulse rates simultaneously and subtract the radial from the apical pulse rate. The result is the pulse deficit. Tachycardia and bradycardia are assessed by measuring the pulse rate for 1 minute. A rate of more than 100 beats per minute is categorized as tachycardia, whereas a rate of less than 60 beats per minute constitutes bradycardia.

The nurse's documentation indicates that a client has a pulse deficit of 14 beats. The pulse deficit is measured by: A) Subtracting 60 (bradycardia) from the client's pulse rate and reporting the difference B) Subtracting the client's pulse rate from 100 (tachycardia) and reporting the difference C) Assessing the apical pulse and the radial pulse for the same minute and subtracting the difference D) Assessing the apical pulse and 30 minutes later assessing the carotid pulse and subtracting the difference

WHEN TO USE STANDARD PRECAUTIONS

Use standard precautions except if an organism is listed. Follow the CDC guidelines If patient presents with S&S of infection ( increase temp, rash, high WBC) isolate immediately until verified. Which patient can you keep at the same room COPD + Negative pressure room = Keep together When to use standard precautions Use standard precautions except if an organism is listed. Follow the CDC guidelines If patient presents with S&S of infection ( increase temp, rash, high WBC) isolate immediately until verified.

Answer: D A normal tympanic membrane is translucent, shiny, and pearly gray. Dark yellow and sticky describes normal moist cerumen (earwax) in front of the tympanic membrane. A white color indicates pus behind the membrane. A pink or red bulging membrane is an indication of inflammation.

Using an otoscope, the nurse can inspect the tympanic membrane. A normal tympanic membrane appears: A) Round and white B) Pink and bulging C) Dark yellow and sticky D) Translucent, shiny, and pearly gray

Answer: C The S2 (dub) sound is the second heart sound and indicates closure of the aortic and pulmonic valves. The closing of the mitral and tricuspid valves is the S1 sound.

While auscultating heart sounds, the nurse documents that S2 is best heard at the base. This sound (S2) correlates with closure of which of the following? A) Aortic and mitral valves B) Mitral and tricuspid valves C) Aortic and pulmonic valves D) Tricuspid and pulmonic valves

Answer: C Complaints of tenderness in the calf during palpation may indicate phlebitis. Other characteristics of phlebitis are swelling, warmth, redness, and sometimes a positive Homans' sign. Cyanosis, pallor, and brown pigmentation around the ankles as well as ulceration and reduced hair growth are indications of venous or arterial insufficiency, which would not cause tenderness on palpation. Venous distention may be indicative of varicosities, which also are not associated with tenderness.

While the nurse was palpating the calf muscles of the client's right leg, the client complained of tenderness. Further assessment by the nurse should include which of the following? A) Observation for reduced hair growth and ulceration B) Observation for venous distention while the client is standing C) Observation of the area for swelling, warmth, redness, and a positive Homans' sign D) Observation for cyanosis, pallor, and change in pigmentation around the ankles

conduction block

a form of regional anesthesia, also known as conduction block.

bed cradle

a frame used to prevent bedclothes from touching all or part of a person's body.

interview

a goal-directed conversation in which one person seeks information from the other.

induration

a hardened place, a lump, as in the skin in a positive reaction to a tuberculin test.

endorphins

a naturally occurring analgesic that the body produces in response to exercise and other stimuli

guided imagery

a process through which the client receives a suggestion that helps control his or her pain or disease. The person learns to visualize himself or herself as powerful and able to conquer pain or disease.

stridor

a shrill and harsh sound (usually refers to the inspiratory sound that occurs when the larynx is obstructed).

pustule

a small elevation of the skin filled with pus or lymph.

wheal

a smooth, slightly elevated skin area, usually pale in the center with are ddened periphery, often accompanied by severe itching when caused by an allergic reaction; small elevation caused by injection of an intra-dermal medication, such as the PPD test for tuberculosis or other skin test

nursing diagnosis

a statement about the client's actual or potential health concerns that can be managed through independent nursing intervention.

specific gravity

a substance's weight, as compared with another. Fluids, such as urine, are compared to (pure water, which has a specific gravity of 1.000) and (Urine 1.010-1.025)

Hemoccult

a test for occult (hidden) blood in stool or body secretions

Hematest

a test for occult (hidden) blood in stool or body secretions.

kyphosis

an abnormal increase in the thoracic curvature of the spine, giving a hunchback appearance, commonly as a result of osteoporosis.

lordosis

an abnormal increase in the thoracic curvature of the spine, giving a hunchback appearance, commonly as a result of osteoporosis.

aphasia

an abnormal neurologic condition in which a person is unable to express oneself through speech or writing.

tumor

an abnormal new tissue growth that has no physiologic use and grows independent of its surrounding structures. May be benign or malignant.

fistula

an abnormal tube-like passage or channel, as an anal fistula or sinus tract.

analgesics

an agent that relieves pain without causing unconsciousness.

data analysis

analyzing each piece of information to determine its relevance to a client's health problems and its relationship to other pieces of information.

spinal anesthesia

anesthetic injected into the subarachnoid space of the spinal cord providing an extensive conduction block. Many types of surgery can be performed in this manner.

disease

any disturbance of a stucture or function of the body

observation

assessment tool that relies on the use of the five senses to discover objective information about the client.

adjuvant

assisting or enhancing therapy given, especially in cancer, to prevent further growth or pain; therapy used which was originally intended for another purpose.

occupied bed

bed holding a client that is unable to get up as a result of his or her condition or generalized weakness.

postoperative bed

bed prepared for a client who is returning from surgery or another procedure that requires transfer into the bed from a stretcher or wheelchair.

open bed

bed that allows linens to be turned down, making it easier for a person to get into or out of.

closed bed

bed used when preparing a unit for a new client—an unoccupied bed

preoperative

before surgery

acute

begins abruptly with marked intensity

ecchymosis

bleeding into the tissue sunder the skin, leaving small bruise.

transfer board

board made of hard plastic used to move patients who are unable to stand from the side of the bed to a chair.

atelectasis

collapse of all or part of a lung.

abscess

collection of pus in a localized area

conjunctivitis

commonly called pinkeye; inflammation of the conjunctiva.

therapeutic communication

communication (usually verbal) with a client that is helpful and beneficial; creating a healing, curative, and safe milieu by using communication.

nonverbal communication

conveying information or messages without speaking or writing. Components include items such as therapeutic touch, gestures, body language, facial expression, and eye contact.

assertiveness

confidence without aggression or passivity, an important skill for a nurse to possess in interpersonal communication.

What should you tell the pt to prepare for the GUAIAC test

don't take anti-inflammatory drugs like Ibuprofen asparin more than 325mg a day 7days prior Colchine Iron Vit C

diplopia

double vision

tympany

drumlike sound over a hollow organ

malaise

feeling of illness; general bodily discomfort.

pain

feeling of suffering, distress or agony, caused by stimulation of specialized nerve endings, a protective device of the body; a subjective sensation (reported by the client).

critical thinking

mix of inquiry, knowledge, intuition, logic, experience, and common sense.

paralysis

motion loss or impairment of sensation in a body part.

What are common side effects of PPE

nausea, diarrhea and fatigue for up to 28 days

flatness

percussion over a muscle causing a soft high pitched flat sound

client reminder device

piece of equipment, most often a vest or a belt, used to ensure the safety of the client (ie, helping client to remain in a chair without falling); also called a client reminder device.

rhonchi

rattling sounds in the throat that resemble snoring (singular, rhonchus).

apical-radial pulse

reading done by measuring both the apical and radial pulses simultaneously, used when it is suspected that the heart is not effectively pumping blood.

erythema

redness

percussion

tapping a body part with short sharp blows to elicit sounds or vibrations that aid in diagnosis; often refers to the use of a percussion hammer to elicit a reflex.

Celsius

temperature scale in which water boils at 100 degrees and freezes at zero (formerly centigrade). "Normal" oral body temperature is 37 degrees Celsius. Celsius scale most often used in health care facilities.

intractable pain

that which cannot be relieved; continuous, relentless, as in intractable pain.

palpation

the act of feeling with the hand, placing the fingers on the skin to determine the condition of under-lying parts.

general anesthesia

the blockage of all body sensations, causing un-consciousness and loss of reflexes.

center of gravity

the center of one's weight; half of one's body weight is below and half above, and half to the left and half to the right of the center of gravity. This concept is important in body mechanics.

hydrometer

urinometer (used to measure specific gravity of a liquid, such as urine).

splinting

use of a pillow or large towel to provide support along a suture line.

percussion

use of fingertips to tap the body's surfact to produce vibration and sound

body mechanics

use of safe and efficient methods of moving and lifting

body mechanics

use of safe and efficient methods of moving and lifting.

footboard

vertical support at the foot of a bed, helps to prevent footdrop.

polyuria

voiding an excessive amount of urine.

urinary frequency

voiding more often than usual without an increase in total urine volume

nursing history

way of soliciting information from the client; may also be called a health interview.

Answer: D Restorative care assists an individual in regaining the maximum possible level of functioning. Home care includes professional and paraprofessional services that are rendered in the home setting. Extended care is intermediate medical or nursing care for individuals with an acute or chronic illness or disability. Assisted care is a setting in which the client is able to function at a higher level of autonomy within a homelike environment but in which care can be given when needed.

A client discharged after suffering a stroke is transferred from a tertiary care facility to another facility for additional care to help the client recover and continue to regain function. This type of care facility is known as: A) Home care B) Assisted care C) Extended care D) Restorative care

Answer: A In the staff model of an MCO, the physicians are salaried employees. In the group model, the MCO contracts with a single group practice. An independent practice association is a group of physicians who are under contact to the organization but are not members of it and whose practices include fee-for-service and capitated clients. The MCO contracts with multiple group practices and/or integrated organizations in the network model.

A client is receiving health care from a health care provider who is a salaried employee. Which model is being followed by the managed care organization (MCO) to which the client belongs? (Select all that apply.) A) Staff model B) Group model C) Network model D) Independent practice association

Answer: D Implementation is the actual delivery of care. Assessment is data gathering. Then the information is developed into a diagnosis and the planning occurs with the diagnosis. Evaluation is the final step of the nursing process.

A client is wheezing and short of breath. The physician orders a medicated nebulizer treatment now and in 4 hours. The nurse is providing what aspect of care? A) Planning B) Evaluation C) Assessment D) Implementation

Answer: C Review of the literature is the first step in the orderly research process to determine what is already known about the problem. Recruiting clients occurs later in the process, after identifying the problem, researching the literature, and designing the study. Experimenting with new nursing procedures that have not been tested or approved is a risk to clients. Surveys are designed to obtain information from large study populations and would not be a first step in the research process.

A clinical nurse develops a better way to secure an intravenous access device in a client and wants to see if it would benefit other clients. The first step in initiating a study should be to: A) Recruit clients to participate in the study. B) Use the new technique and gather client feedback. C) Review current literature related to the clinical problem. D) Survey clients regarding their preferences and feelings regarding the procedure.

Answer: A, B, C, D In this case, all four options are correct. The community health nurse is providing information for the community and helping its members learn to access the help that is available, but not dictating the steps that need to be taken.

A community health nurse is caring for members of a Bosnian community. The nurse determines that the children are undervaccinated and that the community is unaware of this resource. As the nurse assesses the community, the nurse determines that there is a health clinic within 5 miles. The nurse meets with the community leaders and explains the need for immunizations, the location of the clinic, and the process for accessing the health care resources. Which of the following is the nurse doing? (Select all that apply.) A) Improving children's health care B) Teaching the community about illness C) Educating about community resources D) Promoting autonomy in decision making

Answer: C A case manager's competency is defined as the ability to establish an appropriate plan of care that is based on assessment of clients and families and coordinates the provision of needed resources and services across a continuum of care. A collaborator's competency is described as engaging in a combined effort with all those involved in care delivery. A change agent's competency is to implement new and more effective approaches to problems. A client advocate presents the client's point of view so that appropriate resources can be obtained.

A competent community-based nurse must be skilled in fulfilling a variety of roles. The ability to establish an appropriate plan of care that is based on assessment of clients and families and coordinates the provision of needed resources and services across a continuum of care defines the competency of: A) Collaborator B) Change agent C) Case manager D) Client advocate

Answer: B Assessing the learner's needs and readiness to learn are important to increase the success of the learning process. Options A and D are negative responses and would block the learning process. Repeating the old teaching plan is nonproductive and an inefficient application of the nursing process.

A home care nurse educator has repeatedly counseled a 33-year-old male diabetic client concerning the need for dietary compliance. In writing an effective teaching plan the nurse will first: A) Reprimand the client for noncompliant behavior B) Assess the client's learning needs and readiness to learn C) Repeat the old teaching plan to ensure the client's comprehension D) Provide a detailed description of complications associated with the disease process

Answer: D A knowledge-focused trigger is a question regarding new information available on a topic. A problem-focused trigger is one faced while caring for a client or noting a trend. The PICO (population, intervention, comparison, outcome) format is a way to phrase a question to help clarify the question and the parts. A hypothesis is a prediction about the relationship between study variables.

A new nurse on an orthopedic unit is assigned to care for a client undergoing skeletal traction. The nurse asks a colleague, "What is the best practice for cleaning pin sites in skeletal traction?" This question is an example of which of the following? A) Hypothesis B) PICO question C) Problem-focused trigger D) Knowledge-focused trigger

Answer: A, B, C, D All are factors that will impact the client's potential to change.

A nurse at the community clinic nurse cares for a 40-year-old woman who takes insulin to manage diabetes. She is having increasing difficulty controlling the disease, and the nurse wants her to try a new insulin pump to help her manage her diabetes. Which of the following change factors increase the likelihood that she will accept this new insulin pump? (Select all that apply.) A) The innovation or change must be perceived as more advantageous than other alternatives. B) The innovation or change must be compatible with existing needs, values, and past experiences. C) The innovation must be tried on a limited basis. D) Simple innovations or changes are more readily adopted than those that are complex.

Answer: D The homeless person's lack of a storage site for medication and inability to obtain nutritious meals are factors that contribute to poor management of chronic disease. Homeless people are often stereotyped as having a lack of concern for their situations. Poor attire and lack of hygiene are not causes of chronic illness exacerbation. They are signs of the client's status as a member of an at-risk population. It is incorrect for the nurse to assume that the client lacks education and the ability to read.

A nurse is caring for a 64-year-old homeless woman with a chronic respiratory disease in the local community-based clinic. The nurse realizes that the client is at risk of experiencing exacerbation of the disease process related to: A) Poor attire and cleanliness practices B) The client's lack of education and ability to read C) The individual's lack of concern about the disease D) The client's lack of a storage site for medication and the inability to obtain nutritious meals

Answer: B, C The case manager coordinates the efforts of all disciplines to achieve the most efficient and appropriate plan of care for the client, with a focus on discharge planning. Therefore, coordination of transfer to a step-down rehabilitation unit and follow-up after discharge to evaluate that needs have been met are the correct answers.

A nurse is working in an acute care hospital that uses a case management model. About which of the following activities should the nurse communicate with the case manager? (Select all that apply.) A) Management of a client transfer to the radiology department B) Coordination of a client transfer to the step-down rehabilitation unit C) Follow-up after a client's discharge to evaluate whether needs have been met D) Permission for a family to bring in special food for a client

Answer: B Effective qualitative research can be carried out, because through narrative interviews the participants' perceptions can be compared and common characteristics can be discovered. It is difficult to collect data about perceptions or feelings without talking to those involved. Quantitative research involves precise measurements and would not be of use in this study of perceptions. Although obtaining suggestions for possible solutions could be useful, it does not help to identify the problems on this specific unit. The data must be collected first.

A nurse manager is researching the effects of staff shortages on job satisfaction among new graduates. It would be most effective for the nurse to gather data by: A) Directly observing the nursing behaviors on the unit B) Interviewing staff nurses on the unit regarding their perceptions C) Setting up an experimental group and a control group for the study D) Calling on other nurses in the facility to suggest ways of handling the problem

Answer: C Evaluation research is aimed at finding out how well a program, practice, policy, or procedure is working. A survey studies a large group to identify general information, opinions, attitudes, or perceptions. A grounded theory is a theory developed through the collection and analysis of qualitative data. Experimental research collects information about human subjects who are divided into a control group and a comparison group.

A nurse manager wants to determine how well a new policy is working in the clinical area. It would be appropriate to use: A) Survey methods B) Grounded theory C) Evaluation research D) Experimental research

Answer: B A qualitative study involves inductive reasoning to develop generalizations or theories from specific observations or interviews. Historical research establishes facts and relationships concerning past events. Correlational research explores the interrelationships among variables of interest without any intervention by the researcher. An experimental study used tightly controlled subject groups, variables, and procedures to eliminate bias and ensure that findings can be generalized to similar groups of subjects.

A nurse researcher interviews senior oncology nurses, asking them to describe how they deal with the loss of a client. The analysis of the interviews yields common themes describing the nurses' grief. This is an example of which type of study? A) Historical study B) Qualitative study C) Correlational study D) Experimental study

Answer: A Because the clients at the clinic are allowed their choice of the traditional versus the new exercise program, the sampling in this study is not random sampling and can bias study results.

A nurse researcher is designing an exercise study that involves 100 clients who attend a wellness clinic. As the clients come to the clinic, each has a choice as to whether they want to be in the new exercise program or remain in the traditional program. The nurse plans to measure the clients' self-report of exercise before and 6 months after the program begins. What factor might influence the results of this study in an unfavorable way? A) Bias B) Anonymity C) Sample size D) Sampling method

Answer: C The nurse who has held the same position for 2 to 3 years and understands the specific area and client population is termed a competent nurse. The expert is a nurse with diverse experience who can focus on a specific problem and offer multidimensional solutions. The proficient nurse has more than 2 to 3 years' experience and applies knowledge and experience to a situation. The advanced beginner nurse has at least some level of experience.

A nurse who has filled a position on the same unit for 2 years understands the unit's organization and the care of the clients on that nursing unit. Benner defines this nurse as able to anticipate nursing care and to formulate long-range goals; this nurse is given the title: A) Expert nurse B) Proficient nurse C) Competent nurse D) Advanced beginner

Answer: B This process may be carried out with other members of the health care team, and client and family members may be included. All nurses use critical thinking. An advanced care nurse has advanced educational preparation. An evidence-based practitioner draws on research findings as well as clinical expertise and client values. A multidisciplinary practice includes health care members from various fields of activity, such as physical therapy and dietary therapy, along with nursing.

A nurse who uses critical thinking in the decision-making process to provide effective quality care to individuals is known as: A) An advanced care nurse B) A clinical decision maker C) A member of a multidisciplinary practice D) An evidence-based practitioner

Answer: B Quality client care is always the primary focus of nursing practice. Cost control would be a benefit but is not the primary focus. Research is not about technology. Many "old" procedures can be improved through research. Although research is a professional function of nursing, it is not done to serve the profession.

A priority goal for nursing research is: A) Controlling cost for hospitals B) Improving client care C) Keeping up with technological advances D) Maintaining the professional climate in nursing

Answer: D Informed consent means that the research subjects are given full and complete information about the purpose of the study, procedures, data collection, potential harm and benefits, and alternate methods of treatment. Confidentiality rules guarantee that any information the subject provides will not be reported to people outside the research team. Bias is any personal opinion or judgment that may be interjected into the results.

A researcher gives a subject full and complete information about the purpose of a study. This is an example of: A) Bias B) Anonymity C) Confidentiality D) Informed consent

Answer: D Describing, explaining, predicting, and/or prescribing interrelationships among concepts are stated purposes of research.

A theory is a set of concepts, definitions, relationships, and assumptions or propositions to explain a phenomenon. The purposes of the components of a theory are to: A) Describe concepts or connect two concepts that are factual B) Formulate a perceptual experience to describe or label a phenomenon C) Express the global view about the individual, situations, or factors of interest to a specific discipline D) Describe, explain, predict, and/or prescribe interrelationships among the concepts that define the phenomenon

Answer: D Additional training in anesthesia medicine would be required to be a certified registered nurse anesthetist.

An APN is pursuing a job change. Which of the following positions would the APN be unable to fill without meeting additional criteria? A) Case manager B) Nurse manager C) Nurse educator D) Certified registered nurse anesthetist

Answer: C The nurse is assessing the client. Diagnosis occurs after all assessments are completed. Then a plan is developed and implemented. The process is completed with evaluation.

An 18-year-old woman is in the emergency department with fever and cough. The physician asks the nurse to measure vital signs, auscultate lung sounds, listen to heart sounds, determine the level of comfort, and collect blood and sputum samples for analysis. The nurse is performing what aspect of practice? A) Diagnosis B) Evaluation C) Assessment D) Implementation

Answer: D This client has a fever, potentially secondary to the pneumonia previously diagnosed. His blood pressure is within normal limits. His oxygen saturation is at 92%, so this will need to be addressed second. His respiratory rate is high, which can be a result of the fever.

An 82-year-old widower brought via ambulance is admitted to the emergency department with complaints of shortness of breath, anorexia, and malaise. He recently visited his health care provider and was put on an antibiotic for pneumonia. The client indicates that he also takes a diuretic and a beta blocker, which helps his "high blood." Which vital sign value would take priority in initiating care? A) Respiration rate = 20 breaths per minute B) Oxygen saturation by pulse oximetry = 92% C) Blood pressure = 138/84 D) Temperature = 39° C (102° F), tympanic

Answer: A, B, C, D Each of the options is an example of a professional role or responsibility of the professional nurse.

Contemporary nursing requires that the nurse possess knowledge and skills to carry out a variety of professional roles and responsibilities. Examples include which of the following? (Select all that apply.) A) Autonomy and accountability B) Advocacy C) Provision of bedside care D) Health promotion and illness prevention

Answer: A When dealing with clients who are at risk for or may have suffered abuse, it is important to provide protection. Educating the mother on the developmental issues of her infant is important but provides no protection for the victim. Providing protection and eliminating the fear of retribution is a priority upon discovery of abuse. By disregarding the mother's situation, the nurse has failed to intervene for the family in crisis in the community.

During a well-baby visit, the community-based nurse observed patterned bruises and skin abrasions on the face, arms, and throat of the infant's 21-year-old mother. In questioning the mother, the nurse discovers that she is a recent victim of spousal abuse. An important principle in dealing with this client is: A) Ensuring the protection of the mother B) Informing the authorities of the attack C) Educating the mother on well-baby developmental issues D) Continuing with the well-baby examination and disregarding the mother's situation

Answer: C Evidence-based practice is a problem-solving approach to clinical practice that uses the best available evidence, along with the nurse's expertise and the client's preference and values, in making decisions about care. The other answers are incorrect.

Evidence-based practice is defined as: A) Nursing care based on tradition B) Scholarly inquiry embodied in the nursing and biomedical research literature C) A problem-solving approach to clinical practice based on best practices D) Quality nursing care provided in an efficient and economically sound manner

Answer: A The client is the correct choice. The health care facility is where the client goes to receive treatment. The nursing process is how nurses proceed to plan care for the client. Cultural diversity is not the correct choice.

Nursing practice in the twenty-first century is an art and science that is centered on: A) The client B) The nursing process C) Cultural diversity D) The health care facility

Answer: A Phenomena are defined as aspects of reality that can be consciously sensed or experienced.

Nursing theories focus on the phenomena of nursing and nursing care. Which of the following is true of phenomena? A) They are aspects of reality that can be consciously sensed or experienced. B) They convey the general meaning of concepts in a manner that fits the theory. C) They are statements that describe concepts or connect two concepts that are factual. D) They are mental formulations of an object or event that come from individual perceptual experience.

Answer: D Nursing's paradigm includes four linkages: the person, health, environment/situation, and nursing.

Nursing's paradigm includes: A) Health, person, environment, and theory B) Concepts, theory, health, and environment C) Nurses, physicians, models, and client needs D) The person, health, environment/situation, and nursing

Answer: C This is the description of an MCO. In a PPO, choice of care providers is limited to those listed in the group. Medicare is a federally funded national health insurance program. Private insurance is a traditional fee-for-service plan.

The client's health insurance changed, and instead of having a limited number of physicians from whom to choose, the client is voluntarily enrolled in a plan in which medical care is provided by a special group of caregivers. This arrangement is known as: A) Medicare B) Private insurance C) Managed care organization (MCO) D) Preferred provider organization (PPO)

Answer: C Since the "up ad lib" orders are new and the client has been on bed rest, checking orthostatic blood pressure before allowing the client to ambulate is the correct answer. If no sign of orthostatic hypotension is present, then a nursing assistant could assist him to the bathroom. Giving the client a urinal is not a good choice if the client is asymptomatic when orthostatic blood pressure is checked.

The client, who has been on bed rest for 2 days, asks to get out of bed to go to the bathroom. He has new orders for "up ad lib." What action should the nurse take? A) Give him some slippers and tell him where the bathroom is located. B) Ask the nursing assistant to assist him to the bathroom. C) Obtain orthostatic blood pressure measurements. D) Tell him it is not a good idea and provide a urinal.

Answer: C Option 3 defines the focus of community health nursing. Community health nursing focuses on the individual, family, and community. Educational requirements for community-based nurses are not as clearly defined as those for public health nurses. An advanced degree is not always required.

The focus of community health nursing differs from that of public health nursing because the nursing care: A) Is directed at the individual client only B) Is provided by nurses with a graduate degree in community health nursing C) Provides direct care to subpopulations who make up the community as a whole D) Is administered to a collection of individuals who have in common one or more personal or environmental characteristics

Answer: B The Nurse Practice Act regulates the license and practice of nursing; it describes the scope of practice and is the correct answer. The NCLEX-RN national licensure examination is administered in each state to test that candidates have the minimum knowledge level required for practice. Passage of an examination and requirements for certification signify additional knowledge and competence in a specific area. The ANA Congress for Nursing is an organization that addresses legal aspects of nursing practice.

The licensure and practice of nursing is regulated by: A) The NCLEX-RN B) The Nurse Practice Act C) The certification examination D) The ANA Congress for Nursing

Answer: B The apical pulse gives the nurse the most information and accuracy when assessing irregular cardiac rhythm. The carotid or femoral pulses are usually used to assess a client in shock. The radial pulse is adequate for determining routine postoperative vital signs and for checking changes in orthostatic heart rate.

The nurse decides to take an apical pulse instead of a radial pulse. Which of the following client conditions influenced the nurse's decision? A) The client is in shock. B) The client has an arrhythmia. C) The client underwent surgery 18 hours earlier. D) The client showed a response to orthostatic changes.

Answer: C Health promotion includes dietary counseling. Blood glucose monitoring at the pharmacy is an example of illness prevention. Restorative care is care of a client who, for instance, is recovering from complications of diabetes. Any diagnostic procedure or tests completed in the hospital would be examples of such care.

The nurse is giving discharge instructions to a client with newly diagnosed diabetes. The nurse discusses with the client what the dietary intake should be. This is an example of which health care service? A) Tertiary care B) Restorative care C) Health promotion D) Illness prevention

Answer: D Postponing this assessment is definitely a judgment call by the nurse. Postponing is appropriate unless the assessment of respiration is a critical aspect of the test and the client is leaving for the test immediately. Otherwise, it is probably not necessary to invade the client's privacy and disrupt the visitation. Agency policy will dictate whether the respiration rate should be documented as "deferred" or whether documentation can wait until the rate is obtained. Respirations should be counted when the client is "at rest"; therefore, counting respirations during the visitation would not be appropriate. Waiting at the bedside until the visitor leaves is an invasion of privacy for the client and a waste of the nurse's time.

The nurse is to measure vital signs as part of the preparation for a test. The client is talking with a visiting pastor. How should the nurse handle measuring the rate of respiration? A) Count respirations during the time the client is not talking to the visitor. B) Wait at the client's bedside until the visit is over and then count respirations. C) Tell the client it is very important to end the conversation so the nurse can count respirations. D) Document the respiration rate as "deferred" and measure the rate later, since the talking client is obviously not in respiratory distress.

Answer: C The conduct of research must meet ethical standards in which the rights of human subjects are protected. The research participants must be told about the study's purpose and procedure, and their roles in the study. The researcher is always legally responsible for his or her actions. Control of variables is related to the study design, not to informed consent. Confidentiality is part of the ethical nature of research but is not the focus of informed consent.

The nurse researcher obtains informed consent from participants in a study primarily to: A) Release the researcher from legal liability. B) Control variables that might affect the study. C) Ensure that the study subjects understand their roles in the study. D) Maintain the confidentiality of the researcher and the participants.

Answer: A The "Do" step consists of selecting an intervention based on a data review and implementing the change, plus studying the results of the change. The "Plan" step includes reviewing the available data to understand existing practice conditions or problems to identify the need for change. The results of the change are evaluated in the "Study" step. The "Act" step is the incorporation of the findings into current practice.

The nurses on a medical unit have seen an increase in the number of pressure ulcers developing in their clients. The nurses decide to initiate a quality improvement project using the PDSA (plan, do, study, act) model. Which of the following is an example of the "Do" step of that model? A) Implement a new skin care protocol on all medical units. B) Review the data collected on clients cared for using the new protocol. C) Review the quality improvement reports on the six clients who developed ulcers over the previous 3 months. D) Based on findings from clients who developed ulcers, implement an evidence-based skin care protocol.

Answer: A Vulnerable populations are defined as specific populations with unique health care problems. Vulnerable populations are not limited to the very young or older adults. Such individuals are those living in poverty, homeless persons, abused clients, substance abusers, and so on. Members of most vulnerable populations come from different cultures and have different beliefs and values. Vulnerable populations are at risk of experiencing poorer outcomes in response to interventions because of the multiple stressors that affect their daily lives.

Vulnerable populations are more likely to develop health problems. Which of the following is true of these populations? A) They are specific populations with unique health care problems. B) They are limited to the very young and older adult age groups. C) They live in communities with similar cultures, beliefs, and values. D) They frequently experience positive outcomes in response to community health interventions.

Answer: D No individual client assessment should occur in isolation from the environment and conditions of the client's community. Industrial development, types of pollution, and cultural and religious groups are individual elements in the community.

When completing an individual total assessment of a client, the community-based nurse will include consideration of: A) The type of pollution present in the community B) The amount of industrial development in the past 5 years in the community C) The predominant cultural and religious groups found in the community D) The community structures, the population, and the local social system in which the client lives

Answer: B, D Extended care encompasses intermediate medical, nursing, or custodial care for clients recovering from acute illness or clients with chronic illnesses or disabilities. Extended care facilities include intermediate care facilities and skilled nursing facilities.

Which of the following clients should be cared for in an extended care facility with skilled nursing? (Select all that apply.) A) Client who had a stroke, can talk, and has lost bowel and bladder control B) Severely brain injured client on a ventilator who is receiving intravenous medications C) Client with Alzheimer's disease who is abusive, combative, and a threat to self and others D) Young child who recently had a spinal cord injury and is living with quadriplegia and needs to learn a new way of life

Answer: D Shortage of staff could mean less time and personnel to conduct and participate in research. Nursing teams that have teamwork skills can aid research. The desire to change is an incentive for research. Pressure from higher levels in the organization is also an incentive to research.

Which of the following could be a barrier to nursing research? A) Presence of teams in nursing B) Pressure from the administration C) Staff wishes to change a policy D) Shortage of professional nursing staff

Answer: A Day care is an example of respite care because it allows the family to maintain normalcy while the client is under their care. A nursing home client receives 24-hour care in the facility. Home care is an intermittent service in which only certain tasks are performed. Nurse extenders may be hired to perform a specific task, such as bathing.

Which of the following is an example of respite care? A) Day care B) Home care C) Nursing home D) Nurse extender

Answer: C The federal government, which pays for the Medicare and Medicaid programs, is the biggest consumer of health care. The other options are incorrect.

Which of the following is the biggest consumer of health care? A) Hospitals B) Businesses C) Federal government D) Private insurance companies

Answer: A National League for Nursing (NLN) is the correct answer. The master of science in nursing (MSN) degree is earned through advanced educational preparation in nursing. Public Health Administration (PHA) is concerned with areas of public health. The National Institutes of Health (NIH) addresses health on a national level.

Which of the following professional organizations was created to address concerns of members in the nursing profession? A) NLN B) MSN C) PHA D) NIH

Answer: C Prescriptive theories address nursing interventions for a phenomenon and predict the consequence of a specific nursing intervention. Descriptive theories describe the phenomena, speculate on the reason the phenomena occur, and predict nursing phenomena. Grand theories are broad and complex and provide a structural framework for broad, abstract ideas about nursing.

Which of the following statements about prescriptive theories is accurate? A) They describe phenomena. B) They have the ability to explain nursing phenomena. C) They reflect practice and address specific phenomena. D) They provide a structural framework for broad abstract ideas.

Answer: B, C, D Evidence-based practice helps nurses to solve dilemmas in the clinical setting because it combines scientific research with clinical expertise and local values. Evidence-based practice does require nurses to review and critique research and practice findings. Nurses are expected to always meet the standards of practice.

Which of the following statements is true about evidence-based practice? (Select all that apply.) Evidence-based practice: A) Is based only on the results of research B) Assists nurses in meeting standards of practice C) Helps nurses solve dilemmas in the clinical setting D) Requires nurses to review and critique research and practice findings

Answer: D An oxygen saturation of 89% should be addressed first, because this indicates that a client needs oxygen. The high respiratory rate may be a result of hypoxemia and may decrease as the oxygen saturation climbs. The blood pressure is high, but this might be attributed to hypoxemia or anxiety. The heart rate and temperature are within normal limits.

Which of the following values for vital signs would the nurse address first? A) Heart rate = 72 beats per minute B) Respiration rate = 28 breaths per minute C) Blood pressure = 160/86 D) Oxygen saturation by pulse oximetry = 89% E) Temperature = 37.2° C (99° F), tympanic

Answer: C The prospective payment system is one of the most significant factors influencing payment for health care. The prospective payment system groups payments into diagnosis-related groups for Medicare and Medicaid clients. Managed care organizations are systems in which there is administrative control over primary health care services for a defined client population.

Which of the following was most significant in influencing competition in health care costs? A) Medicare and Medicaid B) Diagnosis-related groups C) Prospective payment system D) Managed care organizations

Answer: C Obtaining a sterile specimen requires insertion of a catheter, a procedure that must be performed by a licensed nurse. Therefore, this would not be an appropriate task to delegate to an assistive person. Assistive personnel would be able to ambulate a client, give a bed bath, and add to the I&O record.

Which task is it not appropriate for a professional nurse to delegate to assistive personnel? A) Ambulate a client B) Complete a bed bath C) Obtain a sterile urine specimen D) Complete the intake and output (I&O) record


Kaugnay na mga set ng pag-aaral

4. Prepositions 전치사 - on 착! 달라붙어 있는 ~(위)에

View Set

Chapter 1 Psychological Foundations

View Set

BIOS 1310: Pre-Lab 1: Endocrine System

View Set

Anatomy & Physiology Test Bank Chapter 2

View Set

RLST 3000 Christian Traditions FINAL study guide

View Set

Chapter 11: Fundamentals of the Nervous System

View Set